OB 1

Ace your homework & exams now with Quizwiz!

1. What is the value of the main line fluid rate for your patient, whose total fluid intake is ordered at 150 mL/hour and who is also being given magnesium sulfate at 1 g/hour (1 g = 25 mL/hour) IV piggyback and pitocin at 15 mU/minute (l mU/minute = 1 mL/hour) IV piggyback. ____________

ANS: 110 The rate of infusion of magnesium sulfate (25 mL/hour) and pitocin (15 mL/hour) equals 40 mL/hour. Subtracting the 40 mL from the total ordered of 150 mL leaves 110 mL of main line fluid to be infused per hour.

1. The capacity of the uterus in a term pregnancy is how many times its prepregnant capacity? Record your answer as a whole number._______ times

ANS: 500 The prepregnant capacity of the uterus is about 10 mL, and it reaches 5000 mL (5 L) by the end of the pregnancy, which reflects a 500-fold increase

1. Calculate the estimated date of birth (EDD) in October using Nägele's rule for a patient whose last normal menstrual period (LNMP) began on January 1. Record your answer as a whole number. __________

ANS: 8 Nägele's rule is often used to establish the EDD. This method involves subtracting 3 months from the date that the LNMP began, adding 7 days, and then correcting the year, if appropriate. Subtracting 3 months from January 1 gives you the month of October and adding 7 days = October 8.

27. In a prenatal education class, the nurse is reviewing the importance of using relaxation techniques during labor. Which patient statement will the nurse need to correct? a. "We will practice relaxation techniques only in a quiet setting so I can focus." b. "Relaxation is important during labor because it will help me conserve my energy." c. "If I relax in between contractions, my baby will get more oxygen during labor." d. "My partner and I will practice relaxation throughout the remainder of my pregnancy."

ANS: A Relaxation exercises must be practiced frequently to be useful during labor. Couples begin practice sessions in a quiet, comfortable setting. Later, they practice in other places that simulate the noise and unfamiliar setting of the hospital. The ability to relax during labor is an important component of coping effectively with childbirth. Relaxation conserves energy, decreases oxygen use, and enhances other pain relief techniques. Women learn exercises to help them recognize and release tension. The labor partner assists the woman by providing feedback during exercise sessions and labor.

3. In planning sex education classes for the middle school age group, more emphasis should be placed on a. how to set limits for sexual behavior. b. the inaccuracy of information from peers. c. the use of oral contraceptives to prevent unwanted pregnancy. d. the use of condoms to prevent sexually transmitted diseases as well as pregnancy.

ANS: A Setting limits for sexual behavior is particularly important for younger teenagers who may be pressured to become sexually active before they are physically and emotionally ready. Oral contraceptives are not the preferred method of birth control for teenagers because they forget to take them, and they do not protect against STIs. The use of condoms is appropriate and an important concept to discuss but should not be the emphasis.

16. Which data in the patient's history should the nurse recognize as being pertinent to a possible diagnosis of postpartum depression? a. Previous depressive episode b. Unexpected operative birth c. Ambivalence during the first trimester d. Second pregnancy in a 3-year period

ANS: A A personal or family history of depression or other mental illness is a risk factor for postpartum depression. An operative birth, ambivalence during the first trimester, and two pregnancies in 3 years are not risk factors for postpartum depression.

14. The traditional diet of Asian women includes little meat and few dairy products and may be low in calcium and iron. The nurse can assist a patient increase her intake of these foods by which action? a. Suggest that she eat more tofu, bok choy, and broccoli. b. Suggest that she eat more hot foods during pregnancy. c. Emphasize the need for increased milk intake during pregnancy. d. Tell her husband that she must increase her intake of fruits and vegetables for the baby's sake.

ANS: A The diet should be improved by increasing foods acceptable to the woman. These foods are common in the Asian diet and are good sources of calcium and iron. Pregnancy is considered hot; therefore the woman would eat cold foods. Because milk products are not part of this woman's diet, it should be respected and other alternatives offered. Also, lactose intolerance is common. Fruits and vegetables are cold foods and should be included in the diet. In regard to the family dynamics, however, the husband does not dictate to the wife in this culture.

6. In which step of the nursing process does the nurse determine the appropriate interventions for the identified nursing diagnosis? a. Planning b. Evaluation c. Assessment d. Intervention

ANS: A The third step in the nursing process involves planning care for problems that were identified during assessment. The evaluation phase is determining whether the goals have been met. During the assessment phase, data are collected. The intervention phase is when the plan of care is carried out.

9. The patient makes the statement: "I'm afraid to take the baby home tomorrow." Which response by the nurse would be the most therapeutic? a. "You're afraid to take the baby home?" b. "Don't you have a mother who can come and help?" c. "You should read the literature I gave you before you leave." d. "I was scared when I took my first baby home, but everything worked out."

ANS: A This response uses reflection to show concern and open communication. The other choices are blocks to communication. Asking if the patient has a mother who can come and assist blocks further communication with the patient. Telling the patient to read the literature before leaving does not allow the patient to express her feelings further. Sharing your own birth experience is inappropriate.

17. Which routine nursing assessment is contraindicated for a patient admitted with suspected placenta previa? a. Determining cervical dilation and effacement b. Monitoring FHR and maternal vital signs c. Observing vaginal bleeding or leakage of amniotic fluid d. Determining frequency, duration, and intensity of contractions

ANS: A Vaginal examination of the cervix may result in perforation of the placenta and subsequent hemorrhage. Monitoring FHR and maternal vital signs is a necessary part of the assessment for this patient. Monitoring for bleeding and rupture of membranes is not contraindicated with this patient. Monitoring contractions is not contraindicated with this patient

26. The labor nurse is reviewing breathing techniques with a primiparous patient admitted for induction of labor. When is the best time to encourage the laboring patient to use slow, deep chest breathing with contractions? a. During labor, when she can no longer talk through contractions b. During the first stage of labor, when the contractions are 3 to 4 minutes apart c. Between contractions, during the transitional phase of the first stage of labor d. Between her efforts to push to facilitate relaxation between contractions

ANS: A Focused breathing techniques should not be used in labor until they are actually needed, which is usually when the woman can no longer walk and talk during a contraction. If breathing techniques are used too early, the woman tends to move through the different techniques too quickly, and she may stop using them. In addition, the use of the more complex breathing patterns in latent labor may increase fatigue. Women should be encouraged to adapt the techniques to their own comfort and needs. Breathing deeply between contractions or pushing can increase the possibility of carbon dioxide retention and make the patient dizzy

16. Which patient could safely be cared for by a certified nurse-midwife? a. Gravida 3, para 2, with no complications b. Gravida 1, para 0, with mild hypertension c. Gravida 2, para 1, with insulin-dependent diabetes d. Gravida 1, para 0, with borderline pelvic measurements

ANS: A A certified nurse-midwife (CNM) cares for women who are at low risk for complications. The CNM would not care for a woman with hypertension. The CNM would not care for a woman with insulin-dependent diabetes. The CNM would not care for a woman with borderline pelvic measurements.

52. The labor nurse is admitting a patient in active labor with a history of genital herpes. On assessment, the patient reports a recent outbreak, and the nurse verifies lesions on the perineum. What is the nurse's next action? a. Ask the patient when she last had anything to eat or drink. b. Take a culture of the lesions to verify the involved organism. c. Ask the patient if she has had unprotected sex since her outbreak. d. Use electronic fetal surveillance to determine a baseline fetal heart rate.

ANS: A A cesarean birth is recommended for women with active lesions in the genital area, whether recurrent or primary, at the time of labor. The patient's dietary intake is needed to prepare for surgery. This patient is in active labor and the fetus is at risk for infection if the membranes rupture. The health care provider needs to be notified, and a cesarean birth needs to be performed as soon as possible. There is no need to validate the infection because the patient is well aware of the symptoms of an active infection. Although transmission to sexual partners is valid information, it is not necessary information in an urgent situation such as depicted in this scenario. Electronic fetal surveillance is the standard of care

15. Which of the following statements highlights the nurse's role as a researcher? a. Reading peer-reviewed journal articles b. Working as a member of the interdisciplinary team to provide patient care c. Helping patient to obtain home care postdischarge from the hospital d. Delegating tasks to unlicensed personnel to allow for more teaching time with patients

ANS: A A nurse in a researcher role should look to improve her or his knowledge base by reading and reviewing evidence-based practice information as found in peer-reviewed journals. Working as a member of the interdisciplinary team to provide patient care indicates that the nurse is working as a collaborator. Helping the patient to obtain home care postdischarge from the hospital indicates that the nurse is working as a patient advocate. Delegating tasks to unlicensed personnel in order to allow for more teaching time with patients indicates that the nurse is working as a manager

25. As the triage nurse in the emergency room, you are reviewing results for the high-risk obstetric patient who is in labor because of traumatic injury experienced as a result of a motor vehicle accident (MVA). You note that the Kleihauer-Betke test is positive. Based on this information, you anticipate that a. immediate birth is required. b. the patient should be transferred to the critical care unit for closer observation. c. RhoGAM should be administered. d. a tetanus shot should be administered.

ANS: A A positive Kleihauer-Betke test indicates that fetal bleeding is occurring in the maternal circulation. This is a serious complication and, because the patient is a trauma victim, it is highly likely that she is experiencing an abruption. Therefore the patient should be delivered as quickly as possible to improve outcomes. There is no evidence to support that RhoGAM should be administered, because we have no information related to Rh factor and/or blood type. Similarly, a tetanus shot is not indicated at this time because there is no evidence of penetrating trauma. The patient should be transferred to the obstetric area for birth, not the critical care unit setting

8. Which laboratory result would be a cause for concern if exhibited by a patient at her first prenatal visit during the second month of her pregnancy? a. Rubella titer, 1:6 b. Platelets, 300,000/mm3 c. White blood cell count, 6000/mm3 d. Hematocrit 38%, hemoglobin 13 g/dL

ANS: A A rubella titer of less than 1:8 indicates a lack of immunity to rubella, a viral infection that has the potential to cause teratogenic effects on fetal development. Arrangements should be made to administer the rubella vaccine after birth during the postpartum period because administration of rubella, a live vaccine, would be contraindicated during pregnancy. Women receiving the vaccine during the postpartum period should be cautioned to avoid pregnancy for 3 months. The lab values for WBCs, platelets, and hematocrit/hemoglobin are within the expected range for pregnant women

6. Which complaint made by a patient at 35 weeks of gestation requires additional assessment? a. Abdominal pain b. Ankle edema in the afternoon c. Backache with prolonged standing d. Shortness of breath when climbing stairs

ANS: A Abdominal pain at 35 weeks gestation may indicate preeclampsia, or abruptio placentae. Ankle edema in the afternoon is a normal finding at this stage of the pregnancy. Backaches while standing is a normal finding in the later stages of pregnancy. Shortness of breath is an expected finding at 35 weeks.

13. Which complication of adolescent pregnancy should the nurse plan to monitor? a. Anemia b. Placenta previa c. Abruptio placenta d. Incompetent cervix

ANS: A Adolescent pregnancies are at increased risk for anemia, nutritional deficiencies, pregnancy-associated hypertension, HIV and other STDs, short interval until next pregnancy, and depression. They do not have a higher incidence of placenta previa, abruptio placentae, or incompetent cervix.

13. In teaching a pregnant adolescent about nutrition, what should the nurse include in the care plan? a. Determine the weight gain needed to meet adolescent growth and add 35 lb. b. Suggest that she does not eat at fast food restaurants to avoid foods of poor nutritional value. c. Realize that most adolescents are unwilling to make dietary changes during pregnancy. d. Emphasize the need to eliminate common teen snack foods because they are too high in fat and sodium.

ANS: A Adolescents should gain in the upper range of the recommended weight gain. They also need to gain weight that would be expected for their own normal growth. Adolescents are willing to make changes; however, they still need to be like their peers. Eliminating fast foods will make her appear different from her peers. She should be taught to choose foods that add needed nutrients. Changes in the diet should be kept at a minimum and snacks should be included. Snack foods can be included in moderation and other foods added to make up for the lost nutrients

23. A 36-year-old divorcee with a successful modeling career finds out that her 18-year-old daughter is expecting her first child. Which is a major factor in determining how this woman will respond to becoming a grandmother? a. Her age b. Her career c. Being divorced d. Age of the daughter

ANS: A Age is a major factor in determining the emotional response of prospective grandparents. Young grandparents may not be happy with the stereotype of grandparents as being old. Career responsibilities may have demands that make the grandparents not as accessible but are not a major factor in determining the woman's response to becoming a grandmother. Being divorced is not a major factor that determines the adaptation of grandparents. The age of the daughter is not a major factor that determines the adaptation of grandparents. The age of the grandparent is a major factor.

16. A patient postdelivery is concerned about getting back to her prepregnancy weight as soon as possible. She had only gained 15 lb during her pregnancy. Which assessment factor would be of concern at her 6-week postpartum checkup? a. Patient has lost 30 lb during the 6-week period prior to her scheduled checkup. b. Patient states that she is eating healthy and limiting intake of processed foods. c. Patient relates increased consumption of fruits and vegetables in her diet postbirth. d. Patient has resumed her usual exercise pattern of walking around the neighborhood for 10 minutes each night.

ANS: A Although a certain amount of weight loss is expected in the postpartum period, the fact that the reported weight loss is double the amount of weight gained during the pregnancy places the patient at risk for malnutrition. Further inquiry is needed. Limiting the intake of processed foods is a healthy dietary alternative to decreasing sodium intake. Increases in fruits and vegetables are a healthy dietary alternative to decrease possible occurrence of hypertension. An exercise program is part of a healthy nutrition approach.

7. Which aspect of fetal diagnostic testing is most important to expectant parents? a. Safety of the fetus b. Duration of the test c. Cost of the procedure d. Physical discomfort caused by the procedure

ANS: A Although all of these are considerations, parents are usually most concerned about the safety of the fetus. Parents are concerned about the duration of the test; however, it is not the greatest concern. The cost of the procedure is important to parents, especially those without third-party payers; but again, this is not the greatest concern. With adequate preparation for the procedure by the nurse physical discomfort can be allayed.

11. A pregnant patient would like to know which foods, other than dairy products, contain the most calcium. Which food group would the nurse recommend? a. Legumes b. Lean meat c. Whole grains d. Yellow vegetables

ANS: A Although dairy products contain the greatest amount of calcium, it can also be found in legumes, nuts, dried fruits, and some dark green leafy vegetables. Lean meats are rich in protein and phosphorus. Whole grains are rich in zinc and magnesium. Yellow vegetables are rich in vitamin A

7. Spontaneous termination of a pregnancy is considered to be an abortion if a. the pregnancy is less than 20 weeks. b. the fetus weighs less than 1000 g. c. the products of conception are passed intact. d. there is no evidence of intrauterine infection.

ANS: A An abortion is the termination of pregnancy before the age of viability (20 weeks). The weight of the fetus is not considered because some fetuses of an older age may have a low birth weight. A spontaneous abortion may be complete or incomplete. A spontaneous abortion may be caused by many problems, one being intrauterine infection.

21. A newly pregnant patient tells the nurse that she has irregular periods and is unsure of when she got pregnant. Scheduling an ultrasound is a standing prescription for the patient's health care provider. When is the best time for the nurse to schedule the patient's ultrasound? a. Immediately b. In 2 weeks c. In 4 weeks d. In 6 weeks

ANS: A An embryo can be seen about 5 to 6 weeks after the last menstrual period. At this time the crown-rump length (CRL) of the embryo is the most reliable measure of gestational age. Fetal viability is confirmed by observation of fetal heartbeat, which is visible when the CRL of the embryo is 5 mm

10. An expectant mother, diagnosed with oligohydramnios, asks the nurse what this condition means for the baby. Which statement should the nurse provide for the patient? a. Oligohydramnios can cause poor fetal lung development. b. Oligohydramnios means that the fetus is excreting excessive urine. c. Oligohydramnios could mean that the fetus has a gastrointestinal blockage. d. Oligohydramnios is associated with fetal central nervous system abnormalities.

ANS: A Because an abnormally small amount of amniotic fluid restricts normal lung development, the fetus may have poor fetal lung development. Oligohydramnios may be caused by a decrease in urine secretion. Excessive amniotic fluid production may occur when the gastrointestinal tract prevents normal ingestion of amniotic fluid. Excessive amniotic fluid production may occur when the fetus has a central nervous system abnormality

9. Which effect is a common response to both smoking and cocaine use in the pregnant patient? a. Vasoconstriction b. Increased appetite c. Increased metabolism d. Changes in insulin metabolism

ANS: A Both smoking and cocaine use cause vasoconstriction, which results in impaired placental blood flow to the fetus. Smoking and cocaine use do not increase appetite, change insulin metabolism, or increase metabolism.

28. A pregnant patient has lactose intolerance. Which recommendation will the nurse provide to best help the patient meet dietary needs for calcium? a. Add foods such as nuts, dried fruit, and broccoli to the diet. b. Consume dairy products but take an over-the-counter anti-gas product. c. Increase the intake of dark leafy vegetables, such as spinach and chard. d. Use powdered milk instead of liquid forms of milk.

ANS: A Calcium is present in legumes, nuts, dried fruits, and broccoli, so these foods can be added to increase calcium intake. Although dark leafy vegetables contain calcium, they also contain oxalates that decrease the availability of calcium. Powdered milk contains lactase, similar to the nondehydrated varieties. Milk products should be avoided by patients with lactose intolerance. Adequate calcium may be obtained from food and supplements. Some patients may be able to tolerate lactose free dairy products

18. A laboratory finding indicative of DIC is one that shows a. decreased fibrinogen. b. increased platelets. c. increased hematocrit. d. decreased thromboplastin time.

ANS: A DIC develops when the blood-clotting factor thromboplastin is released into the maternal bloodstream as a result of placental bleeding. Thromboplastin activates widespread clotting, which uses the available fibrinogen, resulting in a decreased fibrinogen level. The platelet count will decrease. The hematocrit may decrease if bleeding is pronounced. The thromboplastin time is prolonged.

3. The clinic nurse is obtaining a health history on a newly pregnant patient. Which is an indication for fetal diagnostic procedures if present in the health history? a. Maternal diabetes b. Weight gain of 25 lb c. Maternal age older than 30 years d. Previous infant weighing more than 3000 g at birth

ANS: A Diabetes is a risk factor in pregnancy because of possible impairment of placental perfusion. Excessive weight gain is an indication for testing. Normal weight gain is 25 to 35 lb. A maternal age older than 35 years is an indication for testing. Having had another infant weighing more than 4000 g is an indication for testing

8. Which statement best describes the changes that occur during the fetal period of development? a. Maturation of organ systems b. Development of basic organ systems c. Resistance of organs to damage from external agents d. Development of placental oxygen-carbon dioxide exchange

ANS: A During the fetal period, the body systems grow in size and mature in function to allow independent existence after birth. Basic organ systems are developed during the embryonic period. The organs are always at risk for damage from external sources; however, the older the fetus, the more resistant will be the organs. The greatest risk is when the organs are developing. The placental system is complete by week 12, but that is not the best description of the fetal period.

3. What is the purpose of the ovum's zona pellucida? a. Prevents multiple sperm from fertilizing the ovum b. Stimulates the ovum to begin mitotic cell division c. Allows the 46 chromosomes from each gamete to merge d. Makes a pathway for more than one sperm to reach the ovum

ANS: A Fertilization causes the zona pellucida to change its chemical composition so that multiple sperm cannot fertilize the ovum. Mitotic cell division begins when the nuclei of the sperm and ovum unite. Each gamete (sperm and ovum) has only 23 chromosomes; there will be 46 chromosomes when they merge. Once sperm has entered the ovum, the zona pellucida changes to prevent other sperm from entering.

4. What point in the pregnancy is the most accurate time to determine gestational age through ultrasound? a. First trimester b. Second trimester c. Third trimester d. No difference in accuracy among the trimesters

ANS: A Gestational age determination by ultrasonography is increasingly less accurate after the first trimester. Gestational age determination is best done in the first trimester. There is a difference in trimesters when doing a gestational age ultrasonography.

6. When is the earliest interval that chorionic villus sampling (CVS) can be performed during pregnancy? a. 4 weeks b. 8 weeks c. 10 weeks d. 14 weeks

ANS: C CVS is normally performed between 10 and 13 weeks gestation. The fetal villus tissue can be analyzed directly for chromosomal, metabolic, or DNA abnormalities. It is too early to perform CVS at 4 or 8 weeks of pregnancy. The test can no longer be performed a 14 weeks gestation. Results are available within 24 to 48 hours.

24. A high-risk labor patient progresses from preeclampsia to eclampsia. Aggressive management is instituted, and the fetus is delivered via cesarean birth. Which finding in the immediate postoperative period indicates that the patient is at risk of developing HELLP syndrome? a. Platelet count of 50,000/mcL b. Liver enzyme levels within normal range c. Negative for edema d. No evidence of nausea or vomiting

ANS: A HELLP syndrome is characterized by Hemolysis, Elevated Liver enzyme levels, and a Low platelet count. A platelet count of 50,000/mcL indicates thrombocytopenia.

42. Nursing intervention for pregnant patients with diabetes is based on the knowledge that the need for insulin is a. varied depending on the stage of gestation. b. increased throughout pregnancy and the postpartum period. c. decreased throughout pregnancy and the postpartum period. d. should not change because the fetus produces its own insulin.

ANS: A Insulin needs decrease during the first trimester, when nausea, vomiting, and anorexia are a factor. Insulin needs increase during the second and third trimesters, when the hormones of pregnancy create insulin resistance in maternal cells. Insulin needs change during pregnancy

15. A pregnant patient asks the nurse how her baby gets oxygen to breathe. What is the nurse's best response? a. "Oxygen-rich blood is delivered through the umbilical vein to the baby." b. "Take lots of deep breaths because the baby gets all of its oxygen from you." c. "You don't need to be concerned about your baby getting enough oxygen." d. "The baby's lungs are not mature enough to actually breathe, so don't worry."

ANS: A Oxygen-rich blood travels from the mother's circulatory system to the placenta and from the placenta to the umbilical vein (veins carry blood to the heart). From the vein, most of the oxygenated blood travels to the fetal liver or the inferior vena cava. Taking deep breaths can temporarily increase oxygenation but can also lead to increased carbon dioxide retention and dizziness. The patient is asking a normal fetal developmental question often asked by pregnant women. Fetal lungs reach maturity by 37 weeks of gestation, but fetal breathing movements are common. Oxygen transport across lung tissue occurs with the first breath.

4. Which intrapartal assessment should be avoided when caring for a patient with HELLP syndrome? a. Abdominal palpation b. Venous sample of blood c. Checking deep tendon reflexes d. Auscultation of the heart and lungs

ANS: A Palpation of the abdomen and liver could result in a sudden increase in intraabdominal pressure, leading to rupture of the subcapsular hematoma. Assessment of heart and lungs is performed on every patient. Checking reflexes is not contraindicated. Venous blood is checked frequently to observe for thrombocytopenia.

40. Which factor is known to increase the risk of gestational diabetes mellitus? a. Previous birth of large infant b. Maternal age younger than 25 years c. Underweight prior to pregnancy d. Previous diagnosis of type 2 diabetes mellitus

ANS: A Prior birth of a large infant suggests gestational diabetes mellitus. A patient younger than 25 is not at risk for gestational diabetes mellitus. Obesity (>90 kg [198 lb]) creates a higher risk for gestational diabetes. The person with type 2 diabetes mellitus already is a diabetic and will continue to be so after pregnancy. Insulin may be required during pregnancy because oral hypoglycemia drugs are contraindicated during pregnancy.

26. Which guidance related to a healthy diet during pregnancy will the nurse provide to a patient in her 1st trimester? a. "Every day you need to have at least 6 ounces of protein from sources such as meat, fish, eggs, beans, nuts, soybean products, and tofu." b. "High-dose vitamin A supplements will promote optimal vision while preventing a common cause of blindness in neonates c. "Meals such as sushi with a cold deli salad made with raw sprouts combine high-fiber foods with protein sources to meet multiple nutritional needs." d. "Vitamin and mineral supplements can meet your nutrient needs if you have inadequate intake because of nausea or a sensation of fullness."

ANS: A Protein sources include meat, poultry, fish, eggs, legumes (e.g., beans, peas, lentils), nuts, and soybean products such as tofu. Pregnant women need 6 to 6.5 oz of protein daily. Vitamin A can cause fetal anomalies of the bones, urinary tract, and central nervous system when taken in high doses. Pregnant women should avoid raw fish and foods such as cold deli salads and raw sprouts. Supplements do not generally contain protein and calories and may lack many necessary nutrients; therefore they cannot serve as food substitutes.

35. For the patient who delivered at 6:30 AM on January 10, Rho(D) immune globulin (RhoGAM) must be administered prior to a. 6:30 AM on January 13. b. 6:30 PM on January 13. c. 6:30 PM on January 14. d. 6:30 AM on January 15.

ANS: A Rho(D) immune globulin (RhoGAM) must be administered within 72 hours after the birth of an Rh-positive infant. 6:30 PM on January 13, 6:30 PM on January 14, and 6:30 AM on January 15 do not fall within the established timeframe

21. A 32-year-old primigravida is admitted with a diagnosis of ectopic pregnancy. Nursing care is based on which of the following? a. Hemorrhage is the primary concern. b. She will be unable to conceive in the future. c. Bed rest and analgesics are the recommended treatment. d. A D&C will be performed to remove the products of conception.

ANS: A Severe bleeding occurs if the fallopian tube ruptures. If the tube must be removed, the patient's fertility will decrease; however, she will be able to achieve a future pregnancy. The recommended treatment is to remove the pregnancy before hemorrhage occurs. A D&C is done on the inside of the uterine cavity. The ectopic is located within the tubes.

10. When a pregnant woman develops ptyalism, which guidance should the nurse provide? a. Chew gum or suck on lozenges between meals. b. Eat nutritious meals that provide adequate amounts of essential vitamins and minerals. c. Take short walks to stimulate circulation in the legs and elevate the legs periodically. d. Use pillows to support the abdomen and back during sleep.

ANS: A Some women experience ptyalism, or excessive salivation. The cause of ptyalism may be decreased swallowing associated with nausea or stimulation of the salivary glands by the ingestion of starch. Small frequent meals and use of chewing gum and oral lozenges offer limited relief for some women. All other options include recommendations for pregnant women; however, they do not address ptyalism.

9. An expected change during pregnancy is a darkly pigmented vertical midabdominal line. The nurse recognizes this alteration as a. epulis. b. linea nigra. c. melasma. d. striae gravidarum.

ANS: B The linea nigra is a dark pigmented line from the fundus to the symphysis pubis. Epulis refers to gingival hypertrophy. Melasma is a different kind of dark pigmentation that occurs on the face. Striae gravidarum (stretch marks) are lines caused by lineal tears that occur in connective tissue during periods of rapid growth.

8. A patient at 24 weeks of gestation reports that she has a glass of wine with dinner every evening. Which rationale should the nurse provide this patient regarding the necessity to eliminate alcohol intake? a. The fetus is placed at risk for altered brain growth. b. The fetus is at risk for severe nervous system injury. c. The patient will be at risk for abusing other substances as well. d. A daily consumption of alcohol indicates a risk for alcoholism.

ANS: A The brain grows most rapidly in the third trimester and is most vulnerable to alcohol exposure during this time. The major concerns are mental retardation, learning disabilities, high activity level, and short attention span. The risk to the patient for abusing other substances is not the major risk for the infant. It has not been proven that daily consumption of alcohol indicates a risk for alcoholism.

7. Which is the most dangerous effect on the fetus of a patient who smokes cigarettes while pregnant? a. Intrauterine growth restriction b. Genetic changes and anomalies c. Extensive central nervous system damage d. Fetal addiction to the substance inhaled

ANS: A The major consequences of smoking tobacco during pregnancy are low-birth-weight infants, prematurity, and increased perinatal loss. Cigarettes normally will not cause genetic changes or extensive central nervous system damage. Addiction is not a normal concern with the neonate.

39. Which major neonatal complication is carefully monitored after the birth of the infant of a diabetic mother? a. Hypoglycemia b. Hypercalcemia c. Hypoinsulinemia d. Hypobilirubinemia

ANS: A The neonate is at highest risk for hypoglycemia because fetal insulin production is accelerated during pregnancy to metabolize excessive glucose from the mother. At birth, the maternal glucose supply stops, and the neonatal insulin exceeds the available glucose, leading to hypoglycemia. Hypocalcemia is associated with preterm birth, birth trauma, and asphyxia, all common problems of the infant of a diabetic mother. Because fetal insulin production is accelerated during pregnancy, the neonate shows hyperinsulinemia. Excess erythrocytes are broken down after birth, releasing large amounts of bilirubin into the neonate's circulation, which results in hyperbilirubinemia.

1. A pregnant patient's biophysical profile score is 8. The patient asks the nurse to explain the results. What is the nurse's most appropriate response? a. "The test results are within normal limits." b. "Immediate birth by cesarean birth is being considered." c. "Further testing will be performed to determine the meaning of this score." d. "An obstetric specialist will evaluate the results of this profile and, within the next week, will inform you of your options regarding birth."

ANS: A The normal biophysical score ranges from 8 to 10 points if the amniotic fluid volume is adequate. A normal score allows conservative treatment of high-risk patients. Birth can be delayed if fetal well-being is an issue. Scores less than 4 would be investigated, and birth could be initiated sooner than planned. This score is within normal range, and no further testing is required at this time. The results of the biophysical profile are usually available immediately after the procedure is performed.

14. The nurse is assessing a newborn immediately after birth. After assigning the first Apgar score of 9, the nurse notes two vessels in the umbilical cord. What is the nurse's next action? a. Assess for other abnormalities of the infant. b. Note the assessment finding in the infant's chart. c. Notify the health care provider of the assessment finding. d. Call for the neonatal resuscitation team to attend the infant immediately

ANS: A The normal finding in the umbilical cord is two arteries and one vein. Two vessels may indicate other fetal anomalies. Notation of the finding is the appropriate next step when the finding is expected. The health care provider will need to be notified; however, the infant is the nurse's primary concern and must be assessed for abnormalities first. The initial Apgar score is 9, indicating no signs of distress or need of resuscitation.

18. The nurse states to the newly pregnant patient, "Tell me how you feel about being pregnant." Which communication technique is the nurse using with this patient? a. Clarifying b. Paraphrasing c. Reflection d. Structuring

ANS: A The nurse is attempting to follow up and check the accuracy of the patient's message. Paraphrasing is restating words other than those used by the patient. Reflection is verbalizing comprehension of what the patient has said. Structuring takes place when the nurse has set guidelines or set priorities.

24. A patient with a BMI of 32 has a positive pregnancy test. What is the maximum number of pounds that the nurse will advise the patient gain during the pregnancy? a. 20 lb b. 25 lb c. 28 lb d. 40 lb

ANS: A The weight gain for obese women is 5 to 9 kg (11 to 20 lb). A BMI of 30 or higher categorizes the patient as obese. The other options refer to minimal or maximal weight gain for patients in other BMI categories.

49. Examination of a newborn in the birth room reveals bilateral cataracts. Which disease process in the maternal history would likely cause this abnormality? a. Rubella b. Cytomegalovirus (CMV) c. Syphilis d. HIV

ANS: A Transmission of congenital rubella causes serious complications in the fetus that may manifest as cataracts, cardiac defects, microcephaly, deafness, intrauterine growth restriction (IUGR), and developmental delays

1. Which physical characteristics decrease as the fetus nears term? (Select all that apply.) a. Vernix caseosa b. Lanugo c. Port wine stain d. Brown fat e. Eyebrows or head hair

ANS: A, B Both vernix caseosa and lanugo decrease as the fetus reaches term. Port wine stain is a birthmark and, if present, will be exhibited at or shortly after birth. Brown fat in the fetus will be maintained in order to maintain core temperature. Eyebrows and head hair increase as the fetus nears term.

2. Transvaginal ultrasonography is often performed during the first trimester. A 6-week-gestation patient expresses concerns over the necessity for this test. The nurse should explain that this diagnostic test may be necessary to determine which of the following? (Select all that apply.) a. Multifetal gestation b. Bicornuate uterus c. Presence and location of pregnancy d. Amniotic fluid volume e. Presence of ovarian cysts

ANS: A, B, C, E A bicornuate uterus, multifetal gestation, presence of ovarian cysts, and presence and location of pregnancy can be determined by transvaginal ultrasound in the first trimester of pregnancy. This procedure is also used for estimating gestational age, confirming fetal viability, identifying fetal abnormalities or chromosomal defects, and identifying the maternal abnormalities mentioned, as well as fibroids. Amniotic fluid volume is assessed during the second and third trimesters. Conventional ultrasound would be used.

2. Many teens wait until the second or even third trimester to seek prenatal care. The nurse should understand that the reasons behind this delay include which of the following? (Select all that apply.) a. Continuing to deny the pregnancy b. Uncertainty about where to go for care c. Lack of realization that they are pregnant d. A desire to gain control over their situation e. Wanting to hide the pregnancy as long as possible

ANS: A, B, C, E Denying the pregnancy, uncertainty about where to go for care, lack of realization of pregnancy, and wanting to hide the pregnancy are all valid reasons for the teen to delay seeking prenatal care. A desire to gain control is not a reason to delay seeking health care.

3. The nurse is assessing a patient's use of complementary and alternative therapies. Which should the nurse document as an alternative or complementary therapy practice? (Select all that apply.) a. Practicing yoga daily b. Drinking green tea in the morning c. Taking omeprazole (Prilosec) once a day d. Using aromatherapy during a relaxing bath e. Wearing a lower back brace when lifting heavy objects

ANS: A, B, D Complementary and alternative (CAM) therapies can be defined as those systems, practices, interventions, modalities, professions, therapies, applications, theories, and claims that are currently not an integral part of the conventional medical system in North America. Yoga is considered to be a mind-body alternative therapy. Green tea and aromatherapy are biologically based complementary therapies. Prilosec and the use of a lower back brace would be therapies consistent with those used by conventional medicine.

2. The nurse is teaching a breastfeeding patient about substances to avoid while she is breastfeeding. Which substances should the nurse include in the teaching session? (Select all that apply.) a. Caffeine b. Alcohol c. Omega-6 fatty acids d. Appetite suppressants e. Polyunsaturated omega-3 fatty acids

ANS: A, B, D Foods high in caffeine should be limited. Infants of mothers who drink more than two or three cups of caffeinated coffee or the equivalent each day may be irritable or have trouble sleeping. Although the relaxing effect of alcohol was once thought to be helpful to the nursing mother, the deleterious effects of alcohol are too important to consider this suggestion appropriate today. An occasional single glass of an alcoholic beverage may not be harmful, but larger amounts may interfere with the milk-ejection reflex and may be harmful to the infant. Nursing mothers should avoid appetite suppressants, which may pass into the milk and harm the infant. The long-chain polyunsaturated omega-3 and omega-6 fatty acids are present in human milk. Therefore they should be included in the mother's diet during lactation.

2. The nurse is teaching a pregnant patient about signs of possible pregnancy complications. Which should the nurse include in the teaching plan? (Select all that apply.) a. Report watery vaginal discharge. b. Report puffiness of the face or around the eyes. c. Report any bloody show when you go into labor. d. Report visual disturbances, such as spots before the eyes. e. Report any dependent edema that occurs at the end of the day.

ANS: A, B, D Watery vaginal discharge could mean that the membranes have ruptured. Puffiness of the face or around the eyes and visual disturbances may indicate preeclampsia or eclampsia. These three signs should be reported. Bloody show as labor starts may mean the mucus plug has been expelled. One of the earliest signs of labor may be bloody show, which consists of the mucus plug and a small amount of blood. This is a normal occurrence. Up to 70% of women have dependent edema during pregnancy. This is not a sign of a pregnancy complication.

1. Which nursing diagnoses may apply to the childbearing family with special needs? (Select all that apply.) a. Risk for spiritual distress b. Risk for injury c. Readiness for enhanced nutrition d. Ineffective breathing pattern e. Situational low self-esteem

ANS: A, B, E A childbearing family with special needs may be at risk to develop spiritual distress, experience injury, and exhibit situational low self-esteem. There are no supportive data to hypothesize an ineffective breathing pattern and/or readiness for enhanced nutrition

3. Which findings are presumptive signs of pregnancy? (Select all that apply.) a. Quickening b. Amenorrhea c. Ballottement d. Goodell's sign e. Chadwick's sign

ANS: A, B, E Quickening, amenorrhea, and Chadwick's sign are presumptive signs of pregnancy. Ballottement and Goodell's sign are probable signs of pregnancy.

4. The nurse is teaching a pregnant patient about food safety during pregnancy and lactation. Which statements by the patient indicate she understood the teaching? (Select all that apply.) a. "I will limit my intake of shrimp to 12 oz a week." b. "I will avoid the soft cheeses made with unpasteurized milk." c. "I plan to continue to pack my bologna sandwich for lunch." d. "I am glad I can still go to the sushi bar during my pregnancy." e. "I will not eat any swordfish or shark while I am pregnant or nursing."

ANS: A, B, E Statements that indicate the patient understood the teaching are limiting shrimp to 12 oz a week, avoiding soft cheeses, and not eating any swordfish. A bologna sandwich should be avoided unless it is reheated until steaming hot. Raw or undercooked fish should be avoided.

1. The nurse is assessing a patient in her 37th week of pregnancy for the psychological responses commonly experienced as birth nears. Which psychological responses should the nurse expect to evaluate? (Select all that apply.) a. The patient is excited to see her baby. b. The patient has not started to prepare the nursery for the new baby. c. The patient expresses concern about how to know if labor has started. d. The patient and her spouse are concerned about getting to the birth center in time. e. The patient and her spouse have not discussed how they will share household tasks.

ANS: A, C, D As birth nears, the expectant patient will express a desire to see the baby. Most pregnant patients are concerned with their ability to determine when they are in labor. Many couples are anxious about getting to the birth facility in time for the birth. As birth nears, a nesting behavior occurs, which means getting the nursery ready. Not preparing the nursery at this stage is not a response that the nurse should expect to assess. Negotiation of tasks is done during this stage. Discussion regarding the division of household chores is not a response that the nurse should expect to assess at this stage.

3. The nurse is advising a lactose-intolerant pregnant patient about calcium intake. Which calcium sources are approximately equivalent to 1 cup of milk? (Select all that apply.) a. 3/4 cup yogurt b. 1 cup of sherbet c. 1 1/4 oz of hard cheese d. 1 1/4 cups of ice cream e. 3/4 cup of low-fat cottage cheese

ANS: A, C, D Calcium sources approximately equivalent to 1 cup of milk include cup yogurt, ounce of hard cheese, and cups of ice cream. It takes 3 cups of sherbet and cups of low-fat cottage cheese to equal the calcium equivalent of 1 cup of milk

1. A pregnant patient reports that she works in a long-term care setting and is concerned about the impending flu season. She asks about receiving the flu vaccine. As the nurse, you are aware that some immunizations are safe to administer during pregnancy, whereas others are not. Which vaccines could this patient receive? (Select all that apply.) a. Tetanus b. Varicella c. Influenza d. Hepatitis A and B e. Measles, mumps, rubella (MMR)

ANS: A, C, D Inactivated vaccines such as those for tetanus, hepatitis A, hepatitis B, and influenza are safe to administer to women who have a risk for contracting or developing the disease. Immunizations with live virus vaccines such as MMR, varicella (chickenpox), or smallpox are contraindicated during pregnancy because of the possible teratogenic effects on the fetus.

4. The prenatal nurse educator is teaching couples the technique of applying sacral pressure during labor. Which should be included in the teaching session? (Select all that apply.) a. The technique can be combined with heat to the area. b. A jiggling motion should be used while applying the pressure. c. Tennis balls may be used to apply the pressure to the sacral area. d. The pressure against the sacrum should be intermittent during the contraction. e. The hand may be moved slowly or remain positioned directly over the sacrum.

ANS: A, C, E Sacral pressure can be cimbined with thermal stimulation to increase effectiveness. The hand may be moved slowly over the area or remain positioned directly over the sacrum, but pressure should be continuous and firm throughout the contraction. Care should be taken not to jiggle the woman, which may be irritating.

2. The nurse is planning care for a patient in her first trimester of pregnancy. The patient is experiencing nausea and vomiting. Which interventions should the nurse plan to share with this patient? (Select all that apply.) a. Suck on hard candy. b. Take prenatal vitamins in the morning. c. Try some herbal tea to relieve the nausea. d. Drink fluids frequently but separate from meals. e. Eat crackers or dry cereal before arising in the morning.

ANS: A, D, E A patient experiencing nausea and vomiting should be taught to suck on hard candy, drink fluids frequently but separately from meals, and eat crackers, dry toast, or dry cereal before arising in the morning. Prenatal vitamins should be taken at bedtime because they may increase nausea if taken in the morning. Before taking herbal tea, the patient should check with her health care provider

3. Which factors contribute to the presence of edema in the pregnant patient? (Select all that apply.) a. Diet consisting of processed foods b. Hemoconcentration c. Increase in colloid osmotic pressure d. Last trimester of pregnancy e. Decreased venous return

ANS: A, D, E Processed foods, which are high in sodium content, can contribute to edema formation. As the pregnancy progresses, because of the weight of the uterus, compression takes place, leading to decreased venous return and an increase in edema formation. A decrease in colloid osmotic pressure would contribute to edema formation and fluid shifting. Hemodilution would also lead to edema formation.

5. A nurse is conducting prenatal education classes for a group of expectant parents. Which information should the nurse include in her discussion of the purpose of amniotic fluid? (Select all that apply.) a. Cushions the fetus b. Protects the skin of the fetus c. Provides nourishment for the fetus d. Allows for buoyancy for fetal movement e. Maintains a stable temperature for the fetus

ANS: A, D, E The amniotic fluid provides cushioning for the fetus against impacts to the maternal abdomen. It provides a stable temperature and allows room and buoyancy for fetal movement. Vernix caseosa, the cheeselike coating on the fetus, provides skin protection. The placenta provides nourishment for the fetus.

4. The nurse is instructing a patient on how to perform kick counts. Which information should the nurse include in the teaching session? (Select all that apply.) a. Use a clock or timer when performing kick counts. b. Your bladder should be full before performing kick counts. c. Notify your health care provider if you have not felt movement in 24 hours. d. Protocols can provide a structured timetable for concentrating on fetal movements. e. You should lie on your side, place your hands on the largest part of the abdomen, and concentrate on the number of movements felt.

ANS: A, D, E The nurse should instruct the patient to lie on her side, place her hands on the largest part of her abdomen, and concentrate on fetal movements. She should use a clock or timer and record the number of movements felt during that time. Protocols are not essential; however, they may give the patient a more structured timetable for when to concentrate on fetal movements. The bladder does not need to be full for kick counts; it is better to have the patient empty her bladder before beginning the assessment of fetal movements. Further evaluation is recommended if the patient feels no movements in 12 hours; 24 hours is too long before notifying the health care provider

17. A pregnant patient has received the results of her triple-screen testing and it is positive. She provides you with a copy of the test results that she obtained from the lab. What would the nurse anticipate as being implemented in the patient's plan of care? a. No further testing is indicated at this time because results are normal. b. Refer to the physician for additional testing. c. Validate the results with the lab facility. d. Repeat the test in 2 weeks and have the patient return for her regularly scheduled prenatal visit.

ANS: B Additional genetic testing is indicated to provide the patient with treatment options. A positive result on a triple-screen test is considered to be an abnormal finding so the patient should be referred to the physician for additional genetic testing. Validation of the test with a lab facility is not necessary because the patient provided you with a copy of the test results. There is no need to repeat the clinical test because the findings have already been determined.

11. Which physiologic finding is consistent with normal pregnancy? a. Systemic vascular resistance increases as blood pressure decreases. b. Cardiac output increases during pregnancy. c. Blood pressure remains consistent independent of position changes. d. Maternal vasoconstriction occurs in response to increased metabolism.

ANS: B Cardiac output increases during pregnancy as a result of increased stroke volume and heart rate. Systemic vascular resistance decreases while blood pressure remains the same. Maternal blood pressure changes in response to patient positioning. In response to increased metabolism, maternal vasodilation is seen during pregnancy.

12. Which environment can assist a pregnant teen to achieve the task of establishing a stable identity? a. Home schooling b. Alternative education program c. School-based mothers' program d. Continuing mainstream high school classes

ANS: C A school-based mothers' program that provides peer support is important. Home schooling, alternative education, and continuing mainstream high school classes would not provide as much peer support.

6. Which maternal condition always necessitates delivery by cesarean birth? a. Partial abruptio placentae b. Total placenta previa c. Ectopic pregnancy d. Eclampsia

ANS: B In total placenta previa, the placenta completely covers the cervical os. The fetus would die if a vaginal birth occurred. If the patient has stable vital signs and the fetus is alive, a vaginal birth can be attempted. If the fetus has already expired, a vaginal birth is preferred. The most common ectopic pregnancy is a tubal pregnancy, which is usually detected and treated in the first trimester. Labor can be safely induced if the eclampsia is under control.

20. Which complication could occur as a result of percutaneous umbilical blood sampling (PUBS)? a. Postdates pregnancy b. Fetal bradycardia c. Placenta previa d. Uterine rupture

ANS: B PUBS is an invasive test whereby a needle is inserted into the umbilical cord to obtain blood as the basis for diagnostic testing with the guidance of ultrasound technology. The most common complication is fetal bradycardia, which is temporary. PUBS has no effect on extending the gestation of pregnancy, the development of placenta previa, or uterine rupture.

18. A pregnant patient arrives for her first prenatal visit at the clinic. She informs the nurse that she has been taking an additional 400 mcg of folic acid prior to becoming pregnant. Based on the patient's history, she has reached 8 weeks' gestation. Which recommendation would the nurse provide regarding folic acid supplementation? a. Have the patient continue to take 400 mcg folic acid throughout her pregnancy. b. Tell the patient that she no longer has to take additional folic acid because it will be included in her prenatal vitamins. c. Have the patient increase her folic acid intake to 1000 mcg throughout the rest of her pregnancy. d. Schedule the patient to go for an AFP (alpha-fetoprotein) test.

ANS: B Prenatal vitamins include adequate folic acid supplementation, so patients should not take additional supplementation as long as they continue their prenatal vitamins. During pregnancy, the recommendation is to increase the folic acid intake to 600 mcg. 1000 mcg of folic acid would be an excessive dose. The AFP test should be done at 15 to 18 weeks' gestation. This is not clinically indicated because the patient is at 8 weeks' gestation.

8. An abortion when the fetus dies but is retained in the uterus is called a. inevitable. b. missed. c. incomplete. d. threatened.

ANS: B A missed abortion refers to a dead fetus being retained in the uterus. An inevitable abortion means that the cervix is dilating with the contractions. An incomplete abortion means that not all the products of conception were expelled. With a threatened abortion, the patient has cramping and bleeding but not cervical dilation.

19. When reviewing a new patient's birth plan, the nurse notices that the patient will be bringing a doula to the hospital during labor. What does the nurse think that this means? a. The patient will have her grandmother as a support person. b. The patient will bring a paid, trained labor support person with her during labor. c. The patient will have a special video she will play during labor to assist with relaxation. d. The patient will have a bag that contains all the approved equipment that may help with the labor process.

ANS: B A doula is a trained labor support person who is employed by the mother to provide labor support. She gives physical support such as massage, helps with relaxation, and provides emotional support and advocacy throughout labor. A doula is usually not a relative of the woman. A doula is a trained labor support person.

47. A patient has a history of drug use and is screened for hepatitis B during the first trimester. Which action is most appropriate? a. Practice respiratory isolation. b. Plan for retesting during the third trimester. c. Discuss the recommendation to bottle feed her baby. d. Anticipate administering the vaccination for hepatitis B as soon as possible

ANS: B A person who has a history of high-risk behaviors should be rescreened during the third trimester. Hepatitis B is transmitted through blood. The first trimester is too early to discuss feeding methods with a woman in the high-risk category. The vaccine may not have time to affect a person with high-risk behaviors.

16. The results of a contraction stress test (CST) are positive. Which intervention is necessary based on this test result? a. Repeat the test in 1 week so that results can be trended based on this baseline result. b. Contact the health care provider to discuss birth options for the patient. c. Send the patient out for a meal and repeat the test to confirm that the results are valid. d. Ask the patient to perform a fetal kick count assessment for the next 30 minutes and then reassess the patient

ANS: B A positive CST test is an abnormal finding, and the provider should be notified so that birth options can be initiated. A positive CST indicates possible fetal compromise. Intervention should not be delayed by 1 week and results do not have to be trended. Because this is an abnormal result, there is no need to repeat the test. Sending the patient out for a meal will delay treatment options and may interfere with possible birth interventions if anesthesia is needed. Fetal kick count assessment is not needed at this time and will further delay treatment interventions for this abnormal result, which indicates fetal compromise.

11. What does optimal nursing care after an amniocentesis include? a. Pushing fluids by mouth b. Monitoring uterine activity c. Placing the patient in a supine position for 2 hours d. Applying a pressure dressing to the puncture site

ANS: B A risk with amniocentesis is the onset of spontaneous contractions. Hydration is important; however, the woman has not been NPO, so this should not be a problem. The supine position may decrease uterine blood flow; the side-lying position is preferred. Pressure dressings are not necessary.

44. Which instructions should the nurse include when teaching a pregnant patient with Class II heart disease? a. Advise her to gain at least 30 lb. b. Instruct her to avoid strenuous activity. c. Inform her of the need to limit fluid intake. d. Explain the importance of a diet high in calcium.

ANS: B Activity may need to be limited so that cardiac demand does not exceed cardiac capacity. Weight gain should be kept at a minimum with heart disease. Iron and folic acid are important to prevent anemia. Fluid intake is necessary to prevent fluid deficits. Fluid intake should not be limited during pregnancy. The patient may also be put on a diuretic.

23. A pregnant woman of normal weight enters her 13th week of pregnancy. If the patient eats and exercises as directed, what will the nurse anticipate as the ongoing weight gain for the remaining trimesters? a. 0.3 lb every week b. 1 lb every week c. 1.8 lb every week d. 2 lb every week

ANS: B After the first 12 weeks (first trimester), the pregnant woman should gain 0.35 to 0.5 kg (0.8 to 1 lb) per week for the remainder of the pregnancy.

11. The priority nursing intervention when admitting a pregnant patient who has experienced a bleeding episode in late pregnancy is to a. monitor uterine contractions. b. assess fetal heart rate and maternal vital signs. c. place clean disposable pads to collect any drainage. d. perform a venipuncture for hemoglobin and hematocrit levels.

ANS: B Assessment of the fetal heart rate (FHR) and maternal vital signs will assist the nurse in determining the degree of the blood loss and its effect on the patient and fetus. Monitoring uterine contractions is important; however, not the top priority. It is important to assess future bleeding, but the top priority is patient and fetal well-being. The most important assessment is to check patient and fetal well-being. The blood levels can be obtained later.

12. You are performing assessments for an obstetric patient who is 5 months pregnant with her third child. Which finding would cause you to suspect that the patient was at risk? a. Patient states that she doesn't feel any Braxton Hicks contractions like she had in her prior pregnancies. b. Fundal height is below the umbilicus. c. Cervical changes, such as Goodell's sign and Chadwick's sign, are present. d. She has increased vaginal secretions.

ANS: B Based on gestational age (20 weeks), the fundal height should be at the umbilicus. This finding is abnormal and warrants further investigation about potential risk. With subsequent pregnancies, multiparas may not perceive Braxton Hicks contractions as being evident compared with their initial pregnancy. Cervical changes such as Goodell's and Chadwick's signs should be present and are considered a normal finding. Increased vaginal secretions are normal during pregnancy as a result of increased vascularity.

20. A patient taking magnesium sulfate has a respiratory rate of 10 breaths per minute. In addition to discontinuing the medication, which action should the nurse take? a. Increase the patient's IV fluids. b. Administer calcium gluconate. c. Vigorously stimulate the patient. d. Instruct the patient to take deep breaths

ANS: B Calcium gluconate reverses the effects of magnesium sulfate. Increasing the patient's IV fluids will not reverse the effects of the medication. Stimulation will not increase the respirations. Deep breaths will not be successful in reversing the effects of the magnesium sulfate.

10. Which finding would indicate concealed hemorrhage in abruptio placentae? a. Bradycardia b. Hard boardlike abdomen c. Decrease in fundal height d. Decrease in abdominal pain

ANS: B Concealed hemorrhage occurs when the edges of the placenta do not separate. The formation of a hematoma behind the placenta and subsequent infiltration of the blood into the uterine muscle results in a very firm, boardlike abdomen. The patient will have shock symptoms that include tachycardia. The fundal height will increase as bleeding occurs. Abdominal pain may increase significantly.

2. Which clinical intervention is the only known cure for preeclampsia? a. Magnesium sulfate b. Delivery of the fetus c. Antihypertensive medications d. Administration of aspirin (ASA) every day of the pregnancy

ANS: B Delivery of the infant is the only known intervention to halt the progression of preeclampsia. Magnesium sulfate is one of the medications used to treat but not cure preeclampsia. Antihypertensive medications are used to lower the dangerously elevated blood pressures in preeclampsia and eclampsia. Low doses of aspirin (81 mg/day) have been administered to women at high risk for developing preeclampsia. This intervention appears to have little benefit.

21. An expectant patient in her third trimester reports that she developed a strong tie to her baby from the beginning and now is really in tune to her baby's temperament. The nurse interprets this as the development of which maternal task of pregnancy? a. Learning to give of herself b. Developing attachment with the baby c. Securing acceptance of the baby by others d. Seeking safe passage for herself and her baby

ANS: B Developing a strong tie in the first trimester and progressing to be in tune is the process of commitment, attachment, and interconnection with the infant. This stage begins in the first trimester and continues throughout the neonatal period. Learning to give of herself is the task that occurs during pregnancy as the woman allows her body to give space to the fetus. She continues with giving to others in the form of food and presents. Securing acceptance of the baby is a process that continues throughout pregnancy as the woman reworks relationships. Seeking safe passage is the task that ends with birth. During this task, the woman seeks health care and carries out cultural practices.

22. Which type of cutaneous stimulation involves massage of the abdomen? a. Imagery b. Effleurage c. Mental stimulation d. Thermal stimulation

ANS: B Effleurage is massage usually performed on the abdomen during contractions. Imagery exercises enhance relaxation by teaching the woman to imagine herself in a relaxing setting. Mental stimulation is a group of methods to decrease pain by increasing mental stimulation. Thermal stimulation decreases pain by using applications of heat and cold.

3. Which advice to the patient is one of the most effective methods for preventing venous stasis? a. Sit with the legs crossed. b. Rest often with the feet elevated. c. Sleep with the foot of the bed elevated. d. Wear elastic stockings in the afternoon

ANS: B Elevating the feet and legs improves venous return and prevents venous stasis. Sitting with the legs crossed will decrease circulation in the legs and increase venous stasis. Elevating the legs at night may cause pressure on the diaphragm and increase breathing problems. Elastic stockings should be applied before lowering the legs in the morning.

19. A patient who is 7 months pregnant states, "I'm worried that something will happen to my baby." Which is the nurse's best response? a. "Your baby is doing fine." b. "Tell me about your concerns." c. "There is nothing to worry about." d. "The doctor is taking good care of you and your baby."

ANS: B Encouraging the patient to discuss her feelings is the best approach. The nurse should not disregard or belittle the patient's feelings. Responding that your baby is doing fine disregards the patient's feelings and treats them as unimportant. Responding that there is nothing to worry about does not answer the patient's concerns. Saying that the doctor is taking good care of you and your baby is belittling the patient's concerns.

19. For which patient would an L/S ratio of 2:1 potentially be considered abnormal? a. A 38-year-old gravida 2,para 1, who is 38 weeks' gestation b. A 24-year-old gravida 1, para 0, who has diabetes c. A 44-year-old gravida 6, para 5, who is at term d. An 18-year-old gravida 1, para 0, who is in early labor at term

ANS: B Even though an L/S ratio of 2:1 is typically considered to be a normal finding to validate fetal lung maturity prior to 38 weeks' gestation, the result may not be accurate in determining fetal lung maturity if a patient is diabetic.

13. Rh incompatibility can occur if the patient is Rh-negative and the a. fetus is Rh-negative. b. fetus is Rh-positive. c. father is Rh-positive. d. father and fetus are both Rh-negative.

ANS: B For Rh incompatibility to occur, the mother must be Rh-negative and her fetus Rh-positive. If the fetus is Rh-negative, the blood types are compatible and no problems should occur. The father's Rh factor is a concern only as it relates to the possible Rh factor of the fetus. If the fetus is Rh-negative, the blood type with the mother is compatible. The father's blood type does not enter into the problem.

50. Which postpartum patient requires further assessment? a. G4 P4 who has had four saturated pads during the last 12 hours b. G1 P1 with Class II heart disease who complains of frequent coughing c. G2 P2 with gestational diabetes whose fasting blood sugar level is 100 mg/dL d. G3 P2 postcesarean patient who has active herpes lesions on the labia

ANS: B Frequent coughing may be a sign of congestive heart failure in the postpartum patient with heart disease. Four saturated pads in a 4-hour period is acceptable postpartum blood loss, a fasting blood sugar is a normal value, and the patient with identified active herpes does not require further assessment.

19. The nurse is scheduling the next appointment for a healthy primigravida currently at 28 weeks gestation. When will the nurse schedule the next prenatal visit? a. 1 week b. 2 weeks c. 3 weeks d. 4 weeks

ANS: B From 29 to 36 weeks, routine prenatal assessment is every 2 weeks. If the pregnancy is high risk, the patient will see the health care provider more frequently.

15. Which data found on a patient's health history would place her at risk for an ectopic pregnancy? a. Ovarian cyst 2 years ago b. Recurrent pelvic infections c. Use of oral contraceptives for 5 years d. Heavy menstrual flow of 4 days' duration

ANS: B Infection and subsequent scarring of the fallopian tubes prevent normal movement of the fertilized ovum into the uterus for implantation. Ovarian cysts do not cause scarring of the fallopian tubes. Oral contraceptives do not increase the risk for ectopic pregnancies. Heavy menstrual flow of 4 days' duration will not cause scarring of the fallopian tubes, which is the main risk factor for ectopic pregnancies.

22. The nurse is reviewing the procedure for alpha-fetoprotein (AFP) screening with a patient at 16 weeks' gestation. The nurse determines that the patient understands the teaching when she states that will be collected for the initial screening process? a. Urine b. Blood c. Saliva d. Amniotic fluid

ANS: B Initial screening is completed with blood. AFP can be detected in amniotic fluid; however, that procedure is more costly and invasive. Procedures progress from least invasive to most invasive.

24. Which comment made by a new mother to her own mother is most likely to encourage the grandmother's participation in the infant's care? a. "Could you help me with the housework today?" b. "The baby is spitting up a lot. What should I do?" c. "I know you are busy, so I'll get John's mother to help me." d. "The baby has a stomachache. I'll call the nurse to find out what to do."

ANS: B Looking to the grandmother for advice encourages her to become involved in the care of the infant. Housework does not encourage the grandmother to participate in the infant's care. Getting John's mother to help and calling the nurse about advice excludes the grandmother.

14. A hospital has achieved Magnet status. Which indicators would be consistent with this type of certification? a. There is stratification of communication in a directed manner between nursing staff and administration. b. There is increased job satisfaction of nurses, with a lower staff turnover rate. c. Physicians are certified in their respective specialty areas. d. All nurses have baccalaureate degrees and certification in their clinical specialty area

ANS: B Magnet status is a certification offered by the ANCC (American Nurses Credentialing Center) in which hospitals apply based on designated criteria that consider nurse job satisfaction, staff patterns, strength, quality of nursing staff, and open communication. It is not based on physician status. Also, certification is not required for all nurses at this point. The expectation with Magnet status is that nurses will continue to expand their knowledge by earning additional degrees and certification.

2. Which analysis of maternal serum is the best predictor of chromosomal abnormalities in the fetus? a. Biophysical profile b. Multiple-marker screening c. Lecithin-to-sphingomyelin ratio d. Blood type and crossmatch of maternal and fetal serum

ANS: B Maternal serum can be analyzed for abnormal levels of alpha-fetoprotein (AFP), human chorionic gonadotropin (hCG), inhibin A, and estriol. The multiple-marker screening may predict chromosomal defects in the fetus. The biophysical profile is used to evaluate fetal status during the antepartum period. Five variables are used; however, none are concerned with chromosomal problems. The lecithin-to-sphingomyelin ratio is used to determine fetal lung maturity. The blood type and crossmatch will not predict chromosomal defects in the fetus.

43. Which form of heart disease in women of childbearing years usually has a benign effect on pregnancy? a. Cardiomyopathy b. Mitral valve prolapse c. Rheumatic heart disease d. Congenital heart disease

ANS: B Mitral valve prolapse is a benign condition that is usually asymptomatic. Cardiomyopathy produces congestive heart failure during pregnancy. Rheumatic heart disease can lead to heart failure during pregnancy. Some congenital heart diseases will produce pulmonary hypertension or endocarditis during pregnancy.

7. A patient in her first trimester complains of nausea and vomiting. The patient asks, "Why is this happening?" What is the nurse's best response? a. "It is due to an increase in gastric motility." b. "It may be due to changes in hormones." c. "It is related to an increase in glucose levels." d. "It is caused by a decrease in gastric secretions."

ANS: B Nausea and vomiting are believed to be caused by increased levels of hormones, decreased gastric motility, and hypoglycemia. Gastric motility decreases during pregnancy. Glucose levels decrease in the first trimester. Gastric secretions decrease, but this is not the main cause of nausea and vomiting.

14. Use Nägele's rule to determine the EDD (estimated day of birth) for a patient whose last menstrual period started on April 12. a. February 19 b. January 19 c. January 21 d. February 7

ANS: B Nägele's rule subtracts 3 months from the month of the last menstrual period (month 4 - 3 = January) and adds 7 days to the day that the last menstrual period started (April 12 + 7 days = April 19), so the correct answer is January 19 of the following calendar year.

9. An expectant mother says to the nurse, "When my sister's baby was born, it was covered in a cheese-like coating. What is the purpose of this coating?" The correct response by the nurse is to explain that the purpose of vernix caseosa is to a. regulate fetal temperature. b. protect the fetal skin from amniotic fluid. c. promote normal peripheral nervous system development. d. allow the transport of oxygen and nutrients across the amnion.

ANS: B Prolonged exposure to amniotic fluid during the fetal period could result in breakdown of the skin without the protection of the vernix caseosa. The amniotic fluid aids in maintaining fetal temperature. Normal peripheral nervous system development is dependent on the nutritional intake of the mother. The amnion is the inner membrane that surrounds the fetus. It is not involved in the oxygen and nutrient exchange

1. A pregnant patient who abuses cocaine admits to exchanging sex for her drug habit. This behavior puts her at a greater risk for a. postmature birth. b. sexually transmitted diseases. c. hypotension and vasodilation. d. depression of the central nervous system.

ANS: B Sex acts exchanged for drugs place the woman at increased risk for sexually transmitted diseases because of having multiple partners and lack of protection. Premature delivery of the infant is one of the most common problems associated with cocaine use during pregnancy. Cocaine causes hypertension and vasoconstriction. Cocaine is a central nervous system stimulant.

16. A patient who smokes one pack of cigarettes daily has a positive pregnancy test. The nurse will explain that smoking during pregnancy increases the risk of which condition? a. Congenital anomalies b. Death before or after birth c. Neonatal hypoglycemia d. Neonatal withdrawal syndrome

ANS: B Smoking during pregnancy increases the risk for spontaneous abortion, low birth weight, abruptio placentae, placenta previa, preterm birth, perinatal mortality, and SIDS. Smoking does not appear to cause congenital anomalies, hypoglycemia, or withdrawal syndrome.

14. A biophysical profile is performed on a pregnant patient. The following assessments are noted: nonreactive stress test (NST), three episodes of fetal breathing movements (FBMs), limited gross movements, opening and closing of hang indicating the presence of fetal tone, and adequate amniotic fluid index (AFI) meeting criteria. Which answer would be the correct interpretation of this test result? a. A score of 10 would indicate that the results are equivocal. b. A score of 8 would indicate normal results. c. A score of 6 would indicate that birth should be considered as a possible treatment option. d. A score of 9 would indicate reassurance.

ANS: B The biophysical profile is used to assess fetal well-being. Five categories of assessment are used in this combination test: fetal monitoring NST, evaluation of FBMs, gross movements, fetal tone, and calculation of the amniotic fluid index (AFI). A maximum of 2 points is used if criteria are met successfully in each category; thus a score in the range of 8 to 10 indicates a normal or reassuring finding. A score of 6 provides equivocal results and further testing or observation is necessary. A score of 4 or less requires immediate intervention, and birth may be warranted. The provided assessments indicate a score of 8 as the only area that has not met the stated criteria in the NST.

29. Which assessment by the nurse would differentiate a placenta previa from an abruptio placentae? a. Saturated perineal pad in 1 hour b. Pain level 0 on a scale of 0 to 10 c. Cervical dilation at 2 cm d. Fetal heart rate at 160 bpm

ANS: B The classic sign of placenta previa is the sudden onset of painless uterine bleeding, whereas abruptio placentae results in abdominal pain and uterine tenderness; heavy bleeding, cervical dilation, and fetal heart rate of 160 bpm could be associated with both conditions.

48. A patient has tested HIV-positive and has now discovered that she is pregnant. Which statement indicates that she understands the risks of this diagnosis? a. "I know I will need to have an abortion as soon as possible." b. "Even though my test is positive, my baby might not be affected." c. "My baby is certain to have AIDS and die within the first year of life." d. "This pregnancy will probably decrease the chance that I will develop AIDS."

ANS: B The fetus is likely to test positive for HIV in the first 6 months, until the inherited immunity from the mother wears off. Many of these babies will convert to HIV-negative status. With the newer drugs, the risk for infection of the fetus has decreased. Also, the life span of an infected newborn has increased. The pregnancy will increase the chance of converting.

1. When planning a healthy diet with a pregnant patient, what should the nurse's first action be? a. Teach the patient about MyPlate. b. Review the patient's current dietary intake. c. Instruct the patient to limit the intake of fatty foods. d. Caution the patient to avoid large doses of vitamins, especially those that are fat-soluble.

ANS: B The first action should be to assess the patient's current dietary pattern and practices because instruction should be geared to what she already knows and does. Teaching the food guide MyPlate is important but not the first action when planning a diet with a pregnant patient. Limiting intake of fatty foods is important in a pregnant patient's diet; however, not the first action. Caution regarding about excessive fat-soluble vitamins is important; however, not the first action.

31. Which intervention is the priority for the patient diagnosed with an intact tubal pregnancy? a. Assessment of pain level b. Administration of methotrexate c. Administration of Rh immune globulin d. Explanation of the common side effects of the treatment plan

ANS: B The goal of medical management of an intact tube is to preserve the tube and improve the chance of future fertility. Methotrexate (a folic acid antagonist) is used to inhibit cell division and stop growth of the embryo. Assessment of pain level, administration of Rh immune globulin, and explaining common side effects of the treatment plan should be implemented in conjunction with or soon after treatment with methotrexate has begun.

5. A patient in her fifth month of pregnancy asks the nurse, "How many more calories should I be eating daily?" What is the correct response by the nurse? a. 180 more calories a day b. 340 more calories a day c. 452 more calories a day d. 500 more calories a day

ANS: B The increased nutritional needs of pregnancy can be met with an additional 340 calories per day. 180 calories are not enough to meet the increased nutritional needs of pregnancy. 452 calories are more than the recommended calories for pregnancy at this gestation. A patient in her third trimester would increase her energy intake by 452 calories per day. 500 calories are more than the recommended calories for pregnancy.

8. The nurse's role in diagnostic testing is to provide which of the following? a. Advice to the couple b. Information about the tests c. Reassurance about fetal safety d. Assistance with decision making

ANS: B The nurse should provide the couple with all necessary information regarding a procedure so that the couple can make an informed decision. The nurse's role is to inform, not to advice. Ensuring fetal safety is not possible with all the diagnostic tests. To offer this is to give false reassurance to the parents. The nurse can inform the couple about potential problems so they can make an informed decision. Decision making should always lie with the couple involved. The nurse should provide information so that the couple can make an informed decision.

22. A patient who was pregnant had a spontaneous abortion at approximately 4 weeks' gestation. At the time of the miscarriage, it was thought that all products of conception were expelled. Two weeks later, the patient presents at the clinic office complaining of "crampy" abdominal pain and a scant amount of serosanguineous vaginal drainage with a slight odor. The pregnancy test is negative. Vital signs reveal a temperature of 100F, with blood pressure of 100/60 mm Hg, irregular pulse 88 beats/minute (bpm), and respirations, 20 breaths per minute. Based on this assessment data, what does the nurse anticipate as a clinical diagnosis? a. Ectopic pregnancy b. Uterine infection c. Gestational trophoblastic disease d. Endometriosis

ANS: B The patient is exhibiting signs of uterine infection, with elevated temperature, vaginal discharge with odor, abdominal pain, and blood pressure and pulse manifesting as shock-trended vitals. Because the pregnancy test is negative, an undiagnosed ectopic pregnancy and gestational trophoblastic disease are ruled out. There is no supportive evidence to indicate a clinical diagnosis of endometriosis at this time; however, it is more likely that this is an infectious process that must be aggressively treated

19. Which patient is most at risk for a low-birth-weight infant? a. 22-year-old, 60 inches tall, normal prepregnant weight b. 18-year-old, 64 inches tall, body mass index is <18.5 c. 30-year-old, 78 inches tall, prepregnant weight is 15 lb above the norm d. 35-year-old, 75 inches tall, total weight gain in previous pregnancies was 33 lb

ANS: B The patient who has a low prepregnancy weight is associated with preterm labor and low-birth-weight infants. Women who are underweight should gain more during pregnancy to meet the needs of pregnancy as well as their own need to gain weight; patients who have a normal prepregnancy weight, who start pregnancy overweight, or who have a history of excessive weight gain in pregnancy are not at risk for low-birth-weight infants.

37. A labor and birth nurse receives a call from the laboratory regarding a preeclamptic patient receiving an IV infusion of magnesium sulfate. The laboratory technician reports that the patient's magnesium level is 7.6 mg/dL. What is the nurse's priority action? a. Stop the infusion of magnesium. b. Assess the patient's respiratory rate. c. Assess the patient's deep tendon reflexes. d. Notify the health care provider of the magnesium level.

ANS: B The therapeutic serum level for magnesium is 4 to 8 mg/dL although it is elevated in terms of normal lab values. Adverse reactions to magnesium sulfate usually occur if the serum level becomes too high. The most important is CNS depression, including depression of the respiratory center. Magnesium is excreted solely by the kidneys, and the reduced urine output that often occurs in preeclampsia allows magnesium to accumulate to toxic levels in the woman. Frequent assessment of serum magnesium levels, deep tendon reflexes, respiratory rate, and oxygen saturation can identify CNS depression before it progresses to respiratory depression or cardiac dysfunction. Monitoring urine output identifies oliguria that would allow magnesium to accumulate and reach excessive levels. Discontinue magnesium if the respiratory rate is below 12 breaths per minute, a low pulse oximeter level (<95%) persists, or deep tendon reflexes are absent. Additional magnesium will make the condition worse.

7. A pregnant patient asks the nurse if she can double her prenatal vitamin dose because she does not like to eat vegetables. What is the nurse's response regarding the danger of taking excessive vitamins? a. Increases caloric intake b. Has toxic effects on the fetus c. Increases absorption of all vitamins d. Promotes development of pregnancy-induced hypertension (PIH)

ANS: B The use of vitamin supplements in addition to food may increase the intake of some nutrients to doses much higher than the recommended amounts. Overdoses of some vitamins have been linked to fetal defects. Vitamin supplements do not contain calories. Vitamin supplements do not have better absorption than natural vitamins and minerals. There is no relationship between vitamin supplements and PIH.

20. What is the term for the step in maternal role attainment that relates to the woman giving up certain aspects of her previous life? a. Fantasy b. Grief work c. Role playing d. Looking for a fit

ANS: B The woman experiences sadness as she realizes that she must give up certain aspects of her previous self and that she can never go back. This is called grief work. Fantasies allow the woman to try on a variety of possibilities or behaviors. This usually deals with how the child will look and the characteristics of the child. Role playing involves searching for opportunities to provide care for infants in the presence of another person. Looking for a fit is when the woman observes the behaviors of mothers and compares them with her own expectations.

20. Changes in the diet of the pregnant patient who has phenylketonuria would include a. adding foods high in vitamin C. b. eliminating drinks containing aspartame c. restricting protein intake to <20 g a day. d. increasing caloric intake to at least 1800 cal/day.

ANS: B Use of aspartame by women with phenylketonuria can result in fetal brain damage because these women lack the enzyme to metabolize aspartame. Adding vitamin C, restricting protein, and increasing caloric intake are not necessary for the pregnant patient with phenylketonuria.

15. Which of these findings would indicate a potential complication related to renal function during pregnancy? a. Increase in glomerular filtration rate (GFR) b. Increase in serum creatinine level c. Decrease in blood urea nitrogen (BUN) d. Mild proteinuria

ANS: B With pregnancy, one would expect the serum creatinine and BUN levels to decrease. An elevation in the serum creatinine level should be investigated. With pregnancy, the GFR increases because of increased renal blood flow and is thus a normal expected finding. A decrease in the blood urea nitrogen level and mild proteinuria is expected findings in pregnancy.

4. The nurse is explaining fetal circulation to a group of nursing students. Which information should be included in the teaching session? (Select all that apply.) a. After birth the ductus venosus remains open, but the other shunts close. b. The foramen ovale shunts blood from the right atrium to the left atrium. c. The ductus venosus shunts blood from the liver to the inferior vena cava. d. The ductus arteriosus shunts blood from the right ventricle to the left ventricle.

ANS: B, C The foramen ovale shunts oxygenated blood from the right atrium to the left atrium, bypassing the lungs. The ductus venosus shunts oxygenated blood from the liver to the inferior vena cava. All shunts close after birth. The ductus arteriosus shunts blood from the right ventricle to the aorta.

1. The nurse is teaching a patient taking prenatal vitamins how to avoid constipation. Which should the nurse plan to include in the teaching session? (Select all that apply.) a. Advise taking a daily laxative for constipation. b. Recommend a diet high in fruits and vegetables. c. Encourage an increase in fluid consumption during the day. d. Increase the intake of whole grains and whole grain products. e. Suggest increasing the intake of dairy products, especially cheeses.

ANS: B, C, D Common sources of dietary fiber include fruits and vegetables (with skins when possible— apples, strawberries, pears, carrots, corn, potatoes with skins, and broccoli), whole grains, and whole grain products—whole wheat bread, bran muffins, bran cereals, oatmeal, brown rice, and whole wheat pasta. Increased intake of fluids can help prevent constipation. A pregnant patient should not take a daily laxative unless prescribed by her health care provider. Increased intake of dairy products, especially cheese, may increase constipation.

1. In consideration of the historic evolution of maternity care, which treatment options were used over the past century? (Select all that apply.) a. During the nineteenth century, women of privilege were delivered by midwives in a hospital setting. b. Granny midwives received their training through a period of apprenticeship. c. The recognition of improved obstetric outcomes was related to increased usage of hygienic practices. d. A shift to hospital-based births occurred as a result of medical equipment designed to facilitate birth. e. The use of chloroform by midwives led to decreased pain during birth.

ANS: B, C, D Training of granny midwives was done by apprenticeship as opposed to formal medical school training. With the advent of usage of hygienic practices, improved health outcomes were seen with regard to a decrease in sepsis. New equipment such as forceps enabled easier birth. Women of privilege in the nineteenth century delivered at home, attended by a midwife. Chloroform was used by physicians and was not available to midwives.

3. A woman who is 36 weeks pregnant asks the nurse to explain the vibroacoustic stimulator (VAS) test. Which should the nurse include in the response? (Select all that apply.) a. The test is invasive. b. The test uses sound to elicit fetal movements. c. The test may confirm nonreactive nonstress test results. d. The test can only be performed if contractions are present. e. Vibroacoustic stimulation can be repeated at 1-minute intervals up to three times

ANS: B, C, E Also referred to as VAS or acoustic stimulation, the vibroacoustic stimulator (similar to an electronic larynx) is applied to the maternal abdomen over the area of the fetal head. Vibration and sound are emitted for up to 3 seconds and may be repeated. A fetus near term responds by increasing the number of gross body movements, which can be easily seen and felt. The procedure can confirm reassuring NST findings and shorten the length of time necessary to obtain NST data. The test is noninvasive and contractions do not need to be present to perform the test

3. The nurse is planning a prenatal class on fetal development. Which characteristics of prenatal development should the nurse include for a fetus of 24 weeks, based on fertilization age? (Select all that apply.) a. Ear cartilage firm b. Skin wrinkled and red c. Testes descending toward the inguinal rings d. Surfactant production nears mature levels e. Fetal movement becoming progressively more noticeable

ANS: B, C, E A fetus of 24 weeks, based on fertilization age, will have wrinkled and red skin, testes descending toward inguinal rings, and the fetal movement becoming progressively more noticeable. Surfactant production nearing the mature levels does not occur until 32 weeks and ear cartilage is not firm until 38 weeks.

2. Along with gas exchange and nutrient transfer, the placenta produces many hormones necessary for normal pregnancy, including which of the following? (Select all that apply.) a. Insulin b. Estrogen c. Progesterone d. Testosterone e. Human chorionic gonadotropin (hCG)

ANS: B, C, E HCG causes the corpus luteum to persist and produce the necessary estrogens and progesterone for the first 6 to 8 weeks. Estrogens cause enlargement of the woman's uterus and breasts and growth of the ductal system in the breasts and, as term approaches, plays a role in the initiation of labor. Progesterone causes the endometrium to change, providing early nourishment. Progesterone also protects against spontaneous abortion by suppressing maternal reactions to fetal antigens and reduces unnecessary uterine contractions. Other hormones produced by the placenta include hCT, hCA, and a number of growth factors. Insulin and testosterone are not secreted by the placenta.

4. Which congenital defects in a newborn are associated with long-term parenting problems? (Select all that apply.) a. Polydactyl b. Cleft lip and palate c. Ventral septal defect d. Ambiguous genitalia

ANS: B, D Although any defect in a newborn produces extreme concern and anxiety, certain defects are associated with long-term parenting problems. Accepting an infant with facial or genital anomalies is particularly difficult for the family and community. Polydactyl and ventral septal defects are reparable, with good outcomes.

4. The nurse is formulating a nursing care plan for a postpartum patient. Which actions by the nurse indicate use of critical thinking skills when formulating the care plan? (Select all that apply.) a. Using a standardized postpartum care plan b. Determining priorities for each diagnosis written c. Writing interventions from a nursing diagnosis book d. Reflecting and suspending judgment when writing the care plan e. Clustering data during the assessment process according to normal versus abnormal

ANS: B, D, E Critical thinking focuses on appraisal of the way the individual thinks, and it emphasizes reflective skepticism. Determining priorities, reflecting and suspending judgment, and clustering data are actions that indicate the use of critical thinking. Using a standardized care plan and writing interventions from a nursing diagnosis book do not show that reflection about the patient's individual care is being done.

1. The nurse is monitoring a patient with severe preeclampsia who is on IV magnesium sulfate. Which signs of magnesium toxicity should the nurse monitor for? (Select all that apply.) a. Cool, clammy skin b. Altered sensorium c. Pulse oximeter reading of 95% d. Respiratory rate of less than 12 breaths per minute e. Absence of deep tendon reflexes

ANS: B, D, E Signs of magnesium toxicity include the following: Respiratory rate of less than 12 breaths per minute (hospitals may specify a rate <14 breaths per minute) Maternal pulse oximeter reading lower than 95% Absence of deep tendon reflexes Sweating, flushing Altered sensorium (confused, lethargic, slurred speech, drowsy, disoriented) Hypotension Serum magnesium value above the therapeutic range of 4 to 8 mg/dL Cold, clammy skin and a pulse oximeter reading of 95% would not be signs of toxicity.

51. The nurse is reviewing the instructions given to a patient at 24 weeks' gestation for a glucose challenge test (GCT). The nurse determines that the patient understands the teaching when she makes which statement? a. "I have to fast the night before the test." b. "I will drink a sugary solution containing 100 g of glucose." c. "I will have blood drawn at 1 hour after I drink the glucose solution." d. "I should keep track of my baby's movements between now and the test."

ANS: C A GCT is administered between 24 and 28 weeks of gestation, often to low- and high-risk antepartum patients. Fasting is not necessary for a GCT, and the woman is not required to follow any pretest dietary instructions. The woman should ingest 50 g of oral glucose solution, and 1 hour later a blood sample is taken. Fetal surveillance with kick counts is an ongoing evaluation for pregnant women; they should contact their health care provider if there is a noticeable decrease in fetal movement.

34. Fraternal twins are delivered by your Rh-negative patient. Twin A is Rh-positive and twin B is Rh-negative. Prior to administering Rho(D) immune globulin (RhoGAM), the nurse should determine the results of the a. direct Coombs test of twin A. b. direct Coombs test of twin B. c. indirect Coombs test of the mother. d. transcutaneous bilirubin level for both twins.

ANS: C Administration of RhoGAM is based on the results of the indirect Coombs test on the patient. A negative result confirms that the mother has not been sensitized by the positive Rh factor of twin A and that RhoGAM is indicated. A direct Coombs test is a diagnostic test used to determine maternal antibodies in fetal blood and to guide treatment of the newborn when Rh and ABO incompatibilities occur. Transcutaneous bilirubin is a noninvasive measure to determine the level of bilirubin in a newborn.

45. Anti-infective prophylaxis is indicated for a pregnant patient with a history of mitral valve stenosis related to rheumatic heart disease because the patient is at risk of developing a. hypertension. b. postpartum infection. c. bacterial endocarditis. d. upper respiratory infections.

ANS: C Because of vegetations on the leaflets of the mitral valve and the increased demands of pregnancy, the patient is at greater risk of bacterial endocarditis. Pulmonary hypertension may occur with mitral valve stenosis, but anti-infective medications will not prevent it from occurring. Women with cardiac problems must be observed for possible infections during the postpartum period but are not given prophylactic antibiotics to prevent them. Women are not put on prophylactic antibiotics to prevent upper respiratory infections.

22. Which situation best describes a man trying on fathering behaviors? a. Reading books on newborn care b. Spending more time with his siblings c. Coaching a little league baseball team d. Exhibiting physical symptoms related to pregnancy

ANS: C Coaching a little league baseball team shows interaction with children and assuming the behavior and role of a father. This best describes a man trying on the role of being a father. Men do not normally read information that is provided in advance. The nurse should be prepared to present information after the baby is born, when it is more relevant. The man will normally seek closer ties with his father. Exhibiting physical symptoms related to pregnancy is called couvade

27. For the pregnant patient who is a vegan, what combination of foods will the nurse advise to meet the nutritional needs for all essential amino acids? a. Eggs and beans b. Fruits and vegetables c. Grains and legumes d. Vitamin and mineral supplements

ANS: C Combining incomplete plant proteins with other plant foods that have complementary amino acids allows intake of all essential amino acids. Dishes that contain grains (e.g., wheat, rice, corn) and legumes (e.g., garbanzo, navy, kidney, or pinto beans, peas, peanuts) are combinations that provide complete proteins. Eggs are not consumed by vegans. Fruits and vegetables alone will not provide the essential amino acids. Vitamin and mineral supplements do not provide amino acids.

3. The clinic nurse is performing a prenatal assessment on a pregnant patient at risk for preeclampsia. Which clinical sign would not present as a symptom of preeclampsia? a. Edema b. Proteinuria c. Glucosuria d. Hypertension

ANS: C Glucose into the urine is not one of the three classic symptoms of preeclampsia. The first sign noted by the pregnant patient is rapid weight gain and edema of the hands and face. Proteinuria usually develops later than the edema and hypertension. The first indication of preeclampsia is usually an increase in the maternal blood pressure.

19. Which assessment in a patient diagnosed with preeclampsia who is taking magnesium sulfate would indicate a therapeutic level of medication? a. Drowsiness b. Urinary output of 20 mL/hour c. Normal deep tendon reflexes d. Respiratory rate of 10 to 12 breaths per minute

ANS: C Magnesium sulfate is administered for preeclampsia to reduce the risk of seizures from cerebral irritability. Hyperreflexia (deep tendon reflexes above normal) is a symptom of cerebral irritability. If the dosage of magnesium sulfate is effective, reflexes should decrease to normal or slightly below normal levels. Drowsiness is another sign of CNS depression from magnesium toxicity. A urinary output of 20 mL/hour is inadequate output. A respiratory rate of 10 to 12 breaths per minute is too slow and could be indicative of magnesium toxicity.

18. A pregnant woman is scheduled to undergo chorionic villus sampling (CVS) based on genetic family history. Which medication does the nurse anticipate will be administered? a. Magnesium sulfate b. Prostaglandin suppository c. RhoGAM if the patient is Rh-negative d. Betamethasone

ANS: C CVS can increase the likelihood of Rh sensitization if a woman is Rh-negative. There is no indication for magnesium sulfate because it is used to stop preterm labor. There is no indication for administration of a prostaglandin suppository. Betamethasone is given to pregnant women in preterm labor to improve fetal lung maturity.

15. A patient has just acknowledged that she is 20 weeks pregnant and confides to the nurse that she has a daily heroin habit. The nurse discusses treatment options for the patient. Which patient statement requires follow-up? a. "My plan is to visit the outpatient clinic daily for treatment." b. "I will see my health care provider at least every 2 weeks." c. "My baby will not have to go through withdrawal when I take methadone." d. "With oral methadone, my baby and I are at decreased risk of infection."

ANS: C Pregnant women who use heroin are often prescribed an alternative drug such as methadone, a synthetic opiate. Methadone can be taken orally once daily and is long-acting, providing consistent blood levels to decrease the adverse fetal effects of wide swings in blood levels found with heroin use. Methadone also reduces the risk of infections from contaminated needles and drug-seeking behavior, such as prostitution. At therapeutic levels, it does not produce the euphoria or sedation of heroin and allows the woman to have a relatively normal lifestyle. The woman who receives a daily dose of methadone in a drug treatment program is more likely to receive prenatal care. However, the newborn must withdraw from methadone after birth.

17. A primipara patient asks about possible support options for her during the labor process. She is apprehensive that her family members will not be prepared to assist her during this time. Which option would be most effective for this patient? a. Reassure the patient that the labor and birth staff consists of highly trained nurses who are well educated to take care of laboring patients so that should be sufficient. b. Encourage the patient to take prepared childbirth classes with her husband because that should provide the best support by a family member. c. Provide information to the patient about obtaining a doula during the labor process. d. Tell the patient that this is a normal feeling based on fear of the unknown and that it will subside once she starts the labor process.

ANS: C Providing information about a doula addresses the patient's concern because the doula's designated role is to provide support during labor. Although it is true that labor and birth nurses are trained in their specialty, the patient is voicing concern for support so her feelings should not be minimized. Encouraging the patient to take prepared childbirth classes is also important; however, it does not address the patient's concern for support. Because this patient is a primipara, it is normal to have some anxiety over the unknown process of the labor experience but again this response minimizes the patient's concern.

21. When explaining the recommended weight gain to your patient, the nurse's teaching should include which statement? a. "All pregnant women need to gain a minimum of 25 to 35 lb." b. "The fetus, amniotic fluid, and placenta require 15 lb of weight gain." c. "Weight gain in pregnancy is based on the patient's prepregnant body mass index." d. "More weight should be gained in the first and second trimesters and less in the third."

ANS: C Recommendations for weight gain in pregnancy are based on the woman's prepregnancy weight for her height (body mass index). Depending on the prepregnant weight, recommendation for weight gain may be more or less than 25 to 35 lb. The combination of the fetus, amniotic fluid, and placenta averages about 11 lb in the patient who has a normal BMI. Less weight should be gained in the first trimester, when the fetus needs fewer nutrients for growth, and more in the third trimester, when fetal growth is accelerated.

6. Some of the embryo's intestines remain within the umbilical cord during the embryonic period because the a. intestines need this time to grow until week 15. b. nutrient content of the blood is higher in this location. c. abdomen is too small to contain all the organs while they are developing. d. umbilical cord is much larger at this time than it will be at the end of pregnancy

ANS: C The abdominal contents grow more rapidly than the abdominal cavity, so part of their development takes place in the umbilical cord. By 10 weeks, the abdomen is large enough to contain them. The intestines remain within the umbilical cord only until about week 10. Blood supply is adequate in all areas; intestines stay in the umbilical cord for about 10 weeks because they are growing faster than the abdomen. Intestines begin their development within the umbilical cord, but only because the liver and kidneys occupy most of the abdominal cavity, not because of the size of the umbilical cord.

12. What is the term for a nonstress test in which there are two or more fetal heart rate accelerations of 15 or more beats per minute (BPM) with fetal movement in a 20-minute period? a. Positive b. Negative c. Reactive d. Nonreactive

ANS: C The nonstress test (NST) is reactive (normal) when there are two or more fetal heart rate accelerations of at least 15 BPM (each with a duration of at least 15 seconds) in a 20-minute period. A positive result is not used with an NST. The contraction stress test (CST) uses positive as a result term. A negative result is not used with an NST. The CST uses negative as a result term. A nonreactive result means that the heart rate did not accelerate during fetal movement.

8. The nurse is conducting a prenatal nutrition education class for a group of nursing students. Which statement best describes the condition known as pica? a. Iron-deficiency anemia b. Intolerance to milk products c. Ingestion of nonfood substances d. Episodes of anorexia and vomiting

ANS: C The practice of eating substances not normally thought of as food is called pica. Clay, dirt, and solid laundry starch are the substances most commonly ingested. Pica may produce iron-deficiency anemia if proper nutrition is decreased. Intolerance to milk products is termed lactose intolerance. Pica is not related to anorexia and vomiting.

6. Which health concern is most likely to be an issue for the older mother? a. Nutrition and diet planning b. Exercise and fitness c. Having enough rest and sleep d. Effective contraceptive methods

ANS: C The woman who delays childbearing may have unique concerns, one of which is having less energy than younger mothers. The older mother is better off financially and can afford better nutrition. Information about exercise and fitness is readily available. The older mother usually has more financial means to search out effective contraceptive methods.

5. A nurse is explaining to the nursing students working on the antepartum unit how to assess for edema. Which edema assessment score indicates edema of the lower extremities, face, hands, and sacral area? a. +1 b. +2 c. +3 d. +4

ANS: C Edema of the extremities, face, and sacral area is classified as +3 edema. Edema classified as +1 indicates minimal edema of the lower extremities. Marked edema of the lower extremities is +2 edema. Generalized massive edema (+4) includes the accumulation of fluid in the peritoneal cavity.

4. Which action should the nurse take when counseling a teenaged patient who has decided to relinquish her baby for adoption? a. Question her about her feelings regarding adoption. b. Tell her she can always change her mind about adoption. c. Affirm her decision while acknowledging her maturity in making it. d. Ask her if anyone is coercing her into the decision to relinquish her baby.

ANS: C A supportive affirming approach by the nurse will strengthen the patient's resolve and help her appreciate the significance of the event. It is important for the nurse to support and affirm the decision the patient has made. This will strengthen the patient's resolve to follow through. Later the patient should be given an opportunity to express her feelings. Telling her that she can always change her mind about adoption should not be an option after the baby is born and placed with the adoptive parents. It is important that the teenager be treated as an adult, with the assumption that she is capable of making an important decision on her own.

13. Which statement is true regarding the shortage of nurses in the United States? a. There are a larger proportion of younger nurses in the workforce as compared with older nurses. b. As a result of decreased RN-to-patient ratios, there is a decrease in patient mortality in the clinical setting. c. Nursing programs are turning away qualified applicants. d. There are adequate classroom and clinical facilities for training RNs

ANS: C According to an Institute of Medicine (IOM) report, by the year 2020, 80% of new RNs should hold baccalaureate degrees. Despite this need, baccalaureate and master's programs are turning away qualified applicants due to an insufficient number of faculty. There are a larger proportion of older nurses in the work force based on current research by the IOM. Increased nurse-to-patient ratios have resulted in decreased patient mortality in the clinical setting. There are currently numerous limitations of both classroom and clinical facilities necessary to train new nurses adequately.

14. The nurse is seeing a 17-year-old female in the clinic for complaints of acne. The nurse plans on taking advantage of this teachable moment with the teen. Which topics will the nurse include in the teen's teaching plan? a. Smoking habits, folic acid intake, and heart disease b. Hyperlipidemia, distracted driving, and menstrual history c. Sexual activity, contraception, and screening for violence d. Optimum weight, hypothyroidism, and sexually transmitted diseases

ANS: C All the topics mentioned are worthy od dicusion. Sexual activity, contraception, and screening for violence have priority related to the age and gender of the patient. Adolescents are seen by health care providers for a variety of reasons before they become pregnant. Counseling to improve health for a future pregnancy should be offered to them during any health care visit. Smoking cessation, attaining optimum weight, folic acid intake, and screening for violence are topics that should be discussed with all young women so that any future pregnancy has the most positive outcome.

41. Which disease process improves during pregnancy? a. Epilepsy b. Bell's palsy c. Rheumatoid arthritis d. Systemic lupus erythematosus (SLE)

ANS: C Although the reason is unclear, marked improvement is seen with rheumatoid arthritis in pregnancy. Unfortunately relapse occurs within 36 months postpartum. With epilepsy, the effect of pregnancy is variable and unpredictable. Seizures may increase, decrease, or remain the same. Bell's palsy was thought to be the result of infection by a virus three times more common during pregnancy and generally occurring in the third trimester. The patient with SLE can have a normal pregnancy but must be treated as high risk because 50% of all births will be premature. Pregnancy can exacerbate SLE.

5. The primary reason for evaluating alpha-fetoprotein (AFP) levels in maternal serum is to determine whether the fetus has which condition? a. Hemophilia b. Sickle cell anemia c. A neural tube defect d. Abnormal lecithin-to-sphingomyelin ratio

ANS: C An open neural tube allows a high level of AFP to seep into the amniotic fluid and enter the maternal serum. Hemophilia is a genetic defect and is best detected with chromosomal studies, such as chorionic villus sampling or amniocentesis. Sickle cell anemia is a genetic defect and is best detected with chromosomal studies such as chorionic villus sampling or amniocentesis. L/S ratios are determined with an amniocentesis and are usually performed in the third trimester.

1. A patient with preeclampsia is being treated with bed rest and intravenous magnesium sulfate. The drug classification of this medication is a a. diuretic. b. tocolytic. c. anticonvulsant. d. antihypertensive.

ANS: C Anticonvulsant drugs act by blocking neuromuscular transmission and depress the central nervous system to control seizure activity. Diuresis is a therapeutic response to magnesium sulfate. A tocolytic drug slows the frequency and intensity of uterine contractions but is not used for that purpose in this scenario. Decreased peripheral blood pressure is a therapeutic response (side effect) of the anticonvulsant magnesium sulfate.

1. During vital sign assessment of a pregnant patient in her third trimester, the patient complains of feeling faint, dizzy, and agitated. Which nursing intervention is most appropriate? a. Have the patient stand up and retake her blood pressure. b. Have the patient sit down and hold her arm in a dependent position. c. Have the patient turn to her left side and recheck her blood pressure in 5 minutes. d. Have the patient lie supine for 5 minutes and recheck her blood pressure on both arms

ANS: C Blood pressure is affected by positioning during pregnancy. The supine position may cause occlusion of the vena cava and descending aorta. Turning the pregnant woman to a lateral recumbent position alleviates pressure on the blood vessels and quickly corrects supine hypotension. Pressures are significantly higher when the patient is standing. This would cause an increase in systolic and diastolic pressures. The arm should be supported at the same level of the heart. The supine position may cause occlusion of the vena cava and descending aorta, creating hypotension.

5. A patient, gravida 2, para 1, comes for a prenatal visit at 20 weeks of gestation. Her fundus is palpated 3 cm below the umbilicus. This finding is a. appropriate for gestational age. b. a sign of impending complications. c. lower than normal for gestational age. d. higher than normal for gestational age.

ANS: C By 20 weeks, the fundus should reach the umbilicus. The fundus should be at the umbilicus at 20 weeks, so 3 cm below the umbilicus is an inappropriate height and needs further assessment. This is lower than ecpected at this date. It may be a complication, but it may also be because of incorrect dating of the pregnancy.

32. Which finding in the exam of a patient with a diagnosis of threatened abortion would change the diagnosis to inevitable abortion? a. Presence of backache b. Rise in hCG level c. Clear fluid from vagina d. Pelvic pressure

ANS: C Clear fluid from the vagina indicates rupture of the membranes. Abortion is usually inevitable (cannot be stopped) when the membranes rupture, the presence of backache and pelvic pressure are common symptoms in threatened abortion, and a rise in the hCG level is consistent with a viable pregnancy.

7. A gravida 1 patient at 32 weeks of gestation reports that she has severe lower back pain. What should the nurse's assessment include? a. Palpation of the lumbar spine b. Exercise pattern and duration c. Observation of posture and body mechanics d. Ability to sleep for at least 6 hours uninterrupted

ANS: C Correct posture and body mechanics can reduce lower back pain caused by increasing lordosis. Pregnancy should not cause alterations in the spine. Any assessment for malformation should be done early in pregnancy. Certain exercises can help relieve back pain. Rest is important for overall well-being; however, the primary concern related to back pain is a thorough evaluation of posture and body mechanics.

28. Which finding in the assessment of a patient following an abruption placenta could indicate a major complication? a. Urine output of 30 mL in 1 hour b. Blood pressure of 110/60 mm Hg c. Bleeding at IV insertion site d. Respiratory rate of 16 breaths per minute

ANS: C DIC is a life-threatening defect in coagulation that may occur following abruptio placentae. DIC allows excess bleeding from any vulnerable area such as IV sites, incisions, gums, or nose. A urine output of 30 mL in 1 hour, blood pressure of 110/60 mm Hg, and respiratory rate of 16 breaths per minute are normal findings in a postpartum patient.

11. The nurse is conducting a staff in-service on multifetal pregnancy. Which statement regarding dizygotic twin development should the nurse include in the teaching session? a. Dizygotic twins arise from two fertilized ova and are the same sex. b. Dizygotic twins arise from a single fertilized ovum and are always of the same sex. c. Dizygotic twins arise from two fertilized ova and may be the same sex or different sexes. d. Dizygotic twins arise from a single fertilized ovum and may be the same sex or different sexes.

ANS: C Dizygotic twins arise from two ova that are fertilized by different sperm. They may be the same or different gender, and they may not have similar physical traits. Monozygotic twins are always the same sex. A single fertilized ovum that produces twins is called monozygotic. Dizygotic twins are from two fertilized ova and may or may not be the same sex.

20. Which information is covered by early pregnancy classes offered in the first and second trimesters? a. Methods of pain relief b. The phases and stages of labor c. Coping with common discomforts of pregnancy d. Prebirth and postbirth care of a patient having a cesarean birth

ANS: C Early pregnancy classes focus on the first two trimesters and cover information on adapting to pregnancy, dealing with early discomforts, and understanding what to expect in the months ahead. Methods of pain relief are discused in a childbirth preparation class. The phases and stages of labor are usually covered in a childbirth preparation class. Cesarean birth preparation classes discuss prebirth and postbirth of a patient having a cesarean birth.

30. A patient at 8 weeks' gestation complains to the nurse, "I feel sick almost every morning. And I throw up at least two or three times a week." What is the nurse's best guidance for this patient? a. "Do you like cheese?" b. "Try eating four meals a day instead of three meals a day." c. "Try eating peanut butter on whole wheat bread right before going to bed." d. "If you can eat enough throughout the day, you don't have to worry about being sick."

ANS: C Eating a bedtime protein snack helps maintain glucose levels throughout the night. Cheese is high in fat and can aggravate nausea. Small and frequent meals is the optimal recommendation. Four meals a day would not be ideal for a patient experiencing nausea, she needs to eat more frequently. Consumption is not the patient's stated concern—it is the nausea and vomiting.

2. Expectant parents ask a prenatal nurse educator, "Which setting for childbirth limits the amount of parent-infant interacUtionS?" NWhTich answOer should the nurse provide for these parents in order to assist them in choosing an appropriate birth setting? a. Birth center b. Home birth c. Traditional hospital birth d. Labor, birth, and recovery room

ANS: C In the traditional hospital setting, the mother may see the infant for only short feeding periods, and the infant is cared for in a separate nursery. Birth centers are set up to allow an increase in parent-infant contact. Home births allow the greatest amount of parent-infant contact. The labor, birth, recovery, and postpartum room setting allows for increased parent-infant contact.

27. What is the priority nursing intervention for the patient who has had an incomplete abortion? a. Methylergonovine (Methergine), 0.2 mg IM b. Preoperative teaching for surgery c. Insertion of IV line for fluid replacement d. Positioning of patient in left side-lying position

ANS: C Initial treatment of an incomplete abortion should be focused on stabilizing the patient's cardiovascular state. Methylergonovine would be administered after surgical treatment, preoperative teaching is not a priority until the patient is stabilized, and the left side-lying position provides no benefit to the patient in this situation.

12. Regarding advanced roles of nursing, which statement related to clinical practice is the most accurate? a. Family nurse practitioners (FNPs) can assist with childbirth care in the hospital setting. b. Clinical nurse specialists (CNSs) provide primary care to obstetric patients. c. Neonatal nurse practitioners provide emergency care in the postbirth setting to high-risk infants. d. A certified nurse midwife (CNM) is not considered to be an advanced practice nurse.

ANS: C Neonatal NPs provide care for the high-risk neonate in the birth room and in the neonatal intensive care unit, as needed. FNPs do not participate in childbirth care; however, they can take care of uncomplicated pregnancies and postbirth care outside of the hospital setting. CNSs work in hospital settings but do not provide primary care services to patients. A CNM is an advanced practice nurse who receives additional certification in the specific area of midwifery.

1. Which clinical conditions are associated with increased levels of alpha fetoprotein (AFP)? (Select all that apply.) a. Down syndrome b. Molar pregnancy c. Twin gestation d. Incorrect gestational age assessment of a normal fetus—estimation is earlier in the pregnancy e. Threatened abortion

ANS: C, D, E Elevated APF levels are seen in multiple gestations, underestimation of fetal age, and threatened abortion. Decreased levels are seen in Down syndrome and a molar pregnancy.

4. Which nursing intervention is an independent function of the professional nurse? a. Administering oral analgesics b. Requesting diagnostic studies c. Teaching the patient perineal care d. Providing wound care to a surgical incision

ANS: C Nurses are now responsible for various independent functions, including teaching, counseling, and intervening in nonmedical problems. Interventions initiated by the physician and carried out by the nurse are called dependent functions. Administrating oral analgesics is a dependent function; it is initiated by a physician and carried out by a nurse. Requesting diagnostic studies is a dependent function. Providing wound care is a dependent function; however, the physician prescribes the type of wound care through direct orders or protocol.

30. A blood-soaked peripad weighs 900 g. The nurse would document a blood loss of _______ mL. a. 1800 b. 450 c. 900 d. 90

ANS: C One g equals 1 mL of blood

12. An infant is diagnosed with fetal anemia. Which information would support this clinical diagnosis? a. Presence of excess maternal hormones b. Maternal blood type O-negative, Rh-negative, and infant blood type O-negative, Rh-negative c. Passive immunity d. Rh-negative mother and Rh-positive baby

ANS: C Passive immunity provides temporary protection to the baby based on the transfer of maternal antibodies. Maternal hormones would not lead to a clinical diagnosis of fetal anemia. These blood types and Rh factors are the same; therefore, no antibodies will be created. In this situation, an Rh-negative mother and Rh-positive baby will result in stimulation of antibodies that will stimulate a reaction leading to hemolysis.

17. A patient with an IUD in place has a positive pregnancy test. When planning care, the nurse will base decisions on which anticipated action? a. A therapeutic abortion will need to be scheduled since fetal damage is inevitable. b. Hormonal analyses will be done to determine the underlying cause of the false-positive test result. c. The IUD will need to be removed to avoid complications such as miscarriage or infection. d. The IUD will need to remain in place to avoid injuring the fetus.

ANS: C Pregnancy with an intrauterine device (IUD) in place is unusual; however, it can occur and cause complications such as spontaneous abortion and infection. A therapeutic abortion is not indicated unless infection occur

28. A patient relates a story of how her boyfriend is feeling her aches and pains associated with her pregnancy. She is concerned that her boyfriend is making fun of her concerns. How would you respond to this patient statement? a. Tell her not to worry because it is natural for her boyfriend to make her feel better by identifying with her pregnancy. b. Refer the patient to a psychologist for counseling to deal with this problem because it is clearly upsetting her. c. Explain that her boyfriend may be experiencing couvade syndrome and that this is a normal finding seen with male partners. d. Ask the patient specifically to define her concerns related to her relationship with her boyfriend and suggest methods to stop this type of behavior by her significant other.

ANS: C Provide factual information that will help reduce stress and modify acceptance. Telling her not to worry does not address the possibility that her boyfriend may be experiencing couvade syndrome. The patient is expressing concern but does not have all the facts related to couvade syndrome and requires education, rather than referral. Couvade syndrome is not an abnormal condition and should be treated with acceptance and understanding.

10. What is the best explanation that the nurse can provide to a patient who is concerned that she has "pseudoanemia" of pregnancy? a. Have her write down her concerns and tell her that you will ask the physician to respond once the lab results have been evaluated. b. Tell her that this is a benign self-limiting condition that can be easily corrected by switching to a high-iron diet. c. Inform her that because of the pregnancy, her blood volume has increased, leading to a substantial dilution effect on her serum blood levels, and that most women experience this condition d. Contact the physician and get a prescription for iron pills to correct this condition.

ANS: C Providing factual information based on physiologic mechanisms is the best option. Although having the patient write down her concerns is reasonable, the nurse should not refer this conversation to the physician but rather address the patient's specific concerns. Switching to a high-iron diet will not correct this condition. This physiologic pattern occurs during pregnancy as a result of hemodilution from excess blood volume. Iron medication is not indicated for correction of this condition. There is no need to contact the physician for a prescription

18. The health care provider reports that the primigravida's fundus can be palpated at the umbilicus. Which priority question will the nurse include in the patient's assessment? a. "Have you noticed that it is easier for you to breathe now?" b. "Would you like to hear the baby's heartbeat for the first time?" c. "Have you felt a fluttering sensation in your lower pelvic area yet?" d. "Have you recently developed any unusual cravings, such as for chalk or dirt?"

ANS: C Quickening is the first maternal sensation of fetal movement and is often described as a fluttering sensation. Quickening is detected at approximately 20 weeks in the primigravida and as early as 16 weeks in the multigravida. The fundus is at the umbilicus at 20 weeks' gestation. Lightening is associated with descent of the fetal head into the maternal pelvis and is associated with improved lung expansion. Lightening occurs approximately 2 weeks before birth in the primipara. Fetal heart tones can be detected by Doppler as early as 9 to 12 weeks of gestation. Pica is the craving for nonnutritive substances such as chalk, dirt, clay, or sand. It can develop at any time during pregnancy. It can be associated with malnutrition and the health care provider should monitor the patient's hematocrit/hemoglobin, zinc, and iron levels.

33. What should the nurse recognize as evidence that the patient is recovering from preeclampsia? a. 1+ protein in urine b. 2+ pitting edema in lower extremities c. Urine output >100 mL/hour d. Deep tendon reflexes +2

ANS: C Rapid reduction of the edema associated with preeclampsia results in urinary output of 4 to 6 L/day as interstitial fluids shift back to the circulatory system. 1+ protein in urine and 2+ pitting edema in lower extremities are signs of continuing preeclampsia. Deep tendon reflexes are not a reliable sign, especially if the patient has been treated with magnesium.

4. What is the gravida and para for a patient who delivered triplets 2 years ago and is now pregnant again? a. 2, 3 b. 1, 2 c. 2, 1 d. 1, 3

ANS: C She has had two pregnancies (gravida 2); para refers to the outcome of the pregnancy rather than the number of infants from that pregnancy. She is pregnant now, so that would make her a gravida 2. She is para 1 because she had one pregnancy that progressed to the age of viability

1. An expectant father asks the nurse, "Which part of the mature sperm contains the male chromosome?" What is the correct response by the nurse? a. X-bearing sperm b. The tail of the sperm c. The head of the sperm d. The middle portion of the sperm

ANS: C The head of the sperm contains the male chromosomes that will join the chromosomes of the ovum. If an X-bearing sperm fertilizes the ovum, the baby will be female. The tail of the sperm helps propel the sperm toward the ovum. The middle portion of the sperm supplies energy for the tail's whip-like action

22. Which patient has correctly increased her caloric intake from her recommended pregnancy intake to the amount necessary to sustain breastfeeding in the first 6 postpartum months? a. From 1800 to 2200 calories per day b. From 2000 to 2500 calories per day c. From 2200 to 2530 calories per day d. From 2500 to 2730 calories per day

ANS: C The increased calories necessary for breastfeeding are 500, with 330 calories coming from increased caloric intake and 170 calories from maternal stores. An increase of 230 calories is insufficient for breastfeeding. An increase of 400 and 500 calories is above the recommended amount.

15. In preparing a pregnant patient for a nonstress test (NST), which of the following should be included in the plan of care? a. Have the patient void prior to being placed on the fetal monitor because a full bladder will interfere with results. b. Maintain NPO status prior to testing. c. Position the patient for comfort, adjusting the tocotransducer belt to locate fetal heart rate. d. Have an infusion pump prepared with oxytocin per protocol for evaluation.

ANS: C The nurse must adjust the tocotransducer to find the best location to pick up and record the fetal heart rate. Positioning the patient for comfort during testing is a prime concern. Although a full bladder may affect patient comfort, it will not interfere with testing results. NPO status is not required for an NST. Instead, a pregnant patient should maintain her normal nutritional intake to provide energy to herself and the fetus. An infusion pump with oxytocin is required for a contraction stress test (CST).

38. Which factor is most important in diminishing maternal, fetal, and neonatal complications in a pregnant patient with diabetes? a. Evaluation of retinopathy by an ophthalmologist b. The patient's stable emotional and psychological status c. Degree of glycemic control before and during the pregnancy d. Total protein excretion and creatinine clearance within normal limits

ANS: C The occurrence of complications can be greatly diminished by maintaining normal blood glucose levels before and during the pregnancy. Even nonpregnant diabetics should have an annual eye examination. Assessing a patient's emotional status is helpful. Coping with a pregnancy superimposed on preexisting diabetes can be very difficult for the whole family; however, it is not the top priority. Baseline renal function is assessed with a 24-hour urine collection and does not diminish the patient's risk for complications.

23. A patient at 36 weeks gestation is undergoing a nonstress (NST) test. The nurse observes the fetal heart rate baseline at 135 beats per minute (bpm) and four nonepisodic patterns of the fetal heart rate reaching 160 bpm for periods of 20 to 25 seconds each. How will the nurse record these findings? a. NST positive, nonreassuring b. NST negative, reassuring c. NST reactive, reassuring d. NST nonreactive, nonreassuring

ANS: C The presence of at least three accelerations of at least 15 beats, over at least 15 seconds, over a duration of at least 20 minutes, is considered reactive and reassuring. Nonreactive testing reveals no or fewer accelerations over the same or longer period. The NST test is not recorded as positive or negative.

12. A pregnant woman complains of frequent heartburn. The patient states that she has never had these symptoms before and wonders why this is happening now. The most appropriate response by the nurse is to a. examine her dietary intake pattern and tell her to avoid certain foods. b. tell her that this is a normal finding during early pregnancy and will resolve as she gets closer to term. c. explain to the patient that physiologic changes caused by the pregnancy make her more likely to experience these types of symptoms. d. refer her to her health care provider for additional testing because this is an abnormal finding.

ANS: C The presentation of heartburn is a normal abnormal finding that can occur in pregnant woman because of relaxation of the lower esophageal sphincter as a result of the physiologic effects of pregnancy. Although foods may contribute to the heartburn, the patient is asking why this presentation is occurring, so the nurse should address the cause first. It is independent of gestation. There is no need to refer to the physician at this time because this is a normal abnormal finding. There is no evidence of complications ensuing from this presentation

4. What is the physiologic reason for vascular volume increasing by 40% to 60% during pregnancy? a. Prevents maternal and fetal dehydration b. Eliminates metabolic wastes of the mother c. Provides adequate perfusion of the placenta d. Compensates for decreased renal plasma flow

ANS: C The primary function of increased vascular volume is to transport oxygen and nutrients to the fetus via the placenta. Preventing maternal and fetal dehydration is not the primary reason for the increase in volume. Assisting with pulling metabolic wastes from the fetus for maternal excretion is one purpose of the increased vascular volume Renal plasma flow increases during pregnancy

36. The labor and birth nurse is reviewing the risk factors for placenta previa with a group of nursing students. The nurse determines that the students understood the discussion when they identify which patient being at the highest risk for developing a placenta previa? a. Female fetus, Mexican-American, primigravida b. Male fetus, Asian-American, previous preterm birth c. Male fetus, African-American, previous cesarean birth d. Female fetus, European-American, previous spontaneous abortion

ANS: C The rate of placenta previa is increasing. It is more common in older women, multiparous women, women who have had cesarean births, and women who had suction curettage for an induced or spontaneous abortion. It is also more likely to recur if a woman has had a placenta previa. African or Asian ethnicity also increases the risk. Cigarette smoking and cocaine use are personal habits that add to a woman's risk for a previa. Previa is more likely if the fetus is male. The Mexican-American primipara has no risk factors for developing a placenta previa. The Asian-American multipara has two risk factors for developing a previa. The African-American multipara has three risk factors for developing a previa. The European-American multigravida has one risk factor for developing a placenta previa

5. Which response by the nurse is the most therapeutic when the patient states, "I'm so afraid to have a cesarean birth"? a. "Everything will be OK." b. "Don't worry about it. It will be over soon." c. "What concerns you most about a cesarean birth?" d. "The physician will be in later and you can talk to him."

ANS: C The response, "What concerns you most about a cesarean birth" focuses on what the patient is saying and asks for clarification, which is the most therapeutic response. The response, "Everything will be ok" is belittling the patient's feelings. The response, "Don't worry about it. It will be over soon" will indicate that the patient's feelings are not important. The response, "The physician will be in later and you can talk to him" does not allow the patient to verbalize her feelings when she wishes to do that.

2. One of the assessments performed in the birth room is checking the umbilical cord for blood vessels. Which finding is considered to be within normal limits? a. One artery and one vein b. Two veins and one artery c. Two arteries and one vein d. Two arteries and two veins

ANS: C The umbilical cord contains two arteries and one vein to transport blood between the fetus and the placenta. Any option other than two arteries and one vein is considered abnormal and requires further assessment. Two veins and one artery is abnormal and may indicate an anomaly. Two arteries instead is a normal finding; this infant would require further assessment for anomalies due to the finding of two veins

16. Which finding on a prenatal visit at 10 weeks might suggest a hydatidiform mole? a. Blood pressure of 120/80 mm Hg b. Complaint of frequent mild nausea c. Fundal height measurement of 18 cm d. History of bright red spotting for 1 day weeks ago

ANS: C The uterus in a hydatidiform molar pregnancy is often larger than would be expected on the basis of the duration of the pregnancy. A patient with a molar pregnancy may have early-onset, pregnancy-induced hypertension. Nausea increases in a molar pregnancy because of the increased production of human chorionic gonadotropin (hCG). The history of bleeding is normally described as being of a brownish color.

5. Physiologic anemia often occurs during pregnancy due to a. inadequate intake of iron. b. the fetus establishing iron stores. c. dilution of hemoglobin concentration. d. decreased production of erythrocytes

ANS: C When blood volume expansion is more pronounced and occurs earlier than the increase in red blood cells, the woman will have physiologic anemia, which is the result of dilution of hemoglobin concentration rather than inadequate hemoglobin. Inadequate intake of iron may lead to true anemia. If the woman does not take an adequate amount of iron, true anemia may occur when the fetus pulls stored iron from the maternal system. There is increased production of erythrocytes during pregnancy

2. Which factor is a major barrier to health care for adolescent mothers? a. Health care workers have a positive attitude. b. The hospital or clinic is within walking distance of the girl's home. c. Seeing a different nurse and/or health care provider at every visit. d. The institution is open days, evenings, and Saturday by special arrangement.

ANS: C Whenever possible, the teen should be scheduled to see the same nurses and practitioners for continuity of care. A positive attitude of the health care providers is important in teen pregnancy care. If the hospital or clinic were within walking distance of the girl's home, it would prevent the teen from missing appointments due to transportation problems. If the institution were open days, evenings, and Saturday by special arrangement, this would be helpful for teens who work, go to school, or have other time of day restrictions. Scheduling conflicts are a major barrier to health care.

3. Which characteristics of fetal alcohol syndrome (FAS) should the nurse expect to assess in affected neonates? (Select all that apply.) a. Hydrocephaly b. Low activity c. Epicanthal folds d. Short palpebral fissures e. Flat midface, with a low nasal bridge

ANS: C, D, E Common facial anomalies associated with FAS include microcephaly, short palpebral fissures (the openings between the eyelids), epicanthal folds, flat midface with a low nasal bridge, indistinct philtrum (groove between the nose and upper lip), and a thin upper lip. Microcephaly is present, not hydrocephaly. Central nervous system impairment includes a high activity level, not a low one.

2. Many communities now offer the availability of free-standing birth centers to provide care for low-risk women during pregnancy, birth, and postpartum. When counseling the newly pregnant patient regarding this option, the nurse should be aware that this type of care setting includes which advantages? (Select all that apply.) a. Staffing by lay midwives b. Equipped for obstetric emergencies c. Less expensive than acute care hospitals d. Safe, homelike births in a familiar setting e. Access to follow-up care for 6 weeks postpartum

ANS: C, D, E Patients who are at low risk and desire a safe, homelike birth are very satisfied with this type of care setting. The new mother may return to the birth center for postpartum follow-up care, breastfeeding assistance, and family planning information for 6 weeks postpartum. Because birth centers do not incorporate advanced technologies into their services, costs are significantly less than in a hospital setting. The major disadvantage of this care setting is that these facilities are not equipped to handle obstetric emergencies. Should unforeseen difficulties occur, the patient must be transported by ambulance to the nearest hospital. Birth centers are usually staffed by certified nurse-midwives (CNMs).

5. The nurse is preparing a patient for a nonstress test (NST). Which interventions should the nurse plan to implement? (Select all that apply.) a. Ensure that the patient has a full bladder. b. Plan approximately 15 minutes for the test. c. Have the patient sit in a recliner with the head elevated 45 degrees. d. Apply electronic monitoring equipment to the patient's abdomen. e. Instruct the patient to press an event marker every time she feels fetal movement

ANS: C, D, E The patient may be seated in a reclining chair or have her head elevated at least 45 degrees. The nurse applies external electronic monitoring equipment to the patient's abdomen to detect the fetal heart rate and any contractions. The woman may be given an event marker to press each time she senses movement. Before the NST, the patient should void and her baseline vital signs should be taken. The NST takes about 40 minutes, allowing for most fetal sleep-wake cycles, although the fetus may show a reassuring pattern more quickly or need more time to awaken and become active. Fifteen minutes would not allow enough time to complete the test.

2. The rate of obesity in the United States has reached epidemic proportions. Morbidity and mortality for both the mother and baby are increased in these circumstances. The nurse caring for the patient with an elevated BMI should be cognizant of these potential complications and plan care accordingly. Significant risks include (Select all that apply.) a. Breech presentation b. Ectopic pregnancy c. Birth defects d. Venous thromboembolism e. Postpartum anemia

ANS: C, D, E Maternal complications associated with pregnancy include: Gestational diabetes, preeclampsia, venous thromboembolism, Caesarean delivery, wound infection, respiratory complications, preterm birth, birth trauma and postpartum anemia. Obese women also have an increased risk of spontaneous abortions and stillbirth. Complications for infants of obese mothers have an increased risk of neural tube defects, hydrocephaly, cardiovascular defects, macrosomia, hypoglycemia, and birth injuries from shoulder dystocia.

9. Which of the following is an example of healthy grieving? a. The mother exhibits an absence of crying or expression of feelings. b. The parents do not mention the baby in conversation with family members. c. The mother asks that the baby be taken away from the delivery area quickly. d. While holding the baby, the mother says to her husband, "He has your eyes and nose."

ANS: D Attachment behaviors are necessary for healthy grieving. Absence of crying and not mentioning the baby may be signs of denial. By not seeing the baby, attachment and therefore healthy grieving will not occur.

2. The nurse is meeting with a patient with an elevated BMI regarding an optimal diet for pregnancy. Which major source of nutrients should be a significant component of this patient's diet? a. Fats b. Fiber c. Simple sugars d. Complex carbohydrates

ANS: D Complex carbohydrates supply the pregnant woman with vitamins, minerals, and fiber. Fats provide 9 calories in each gram, in contrast to carbohydrates and proteins, which provide only 4 calories in each gram. Fiber is supplied primarily by complex carbohydrates. The most common simple carbohydrate is table sugar, which is a source of energy but does not provide any nutrients.

9. Which factors should be considered a contraindication for transcervical chorionic villus sampling? a. Rh-negative mother b. Gestation less than 15 weeks c. Maternal age younger than 35 years d. Positive for group B Streptococcus

ANS: D Maternal infection is a risk with this procedure, and it is contraindicated if the patient has an active infection in the cervix, vagina, or pelvic area. This procedure is done between 10 and 12 weeks. This procedure is usually done for women older than 35; however, if the woman is at high risk for fetal anomalies, her age is not a contraindication. The procedure can still be performed; however, Rh sensitization may occur if the mother is Rh-negative. Rho(D) immune globulin can be administered following the procedure.

3. Which statement best describes the advantage of a labor, birth, recovery, and postpartum (LDRP) room? a. The family is in a familiar environment. b. They are less expensive than traditional hospital rooms. c. The infant is removed to the nursery to allow the mother to rest. d. The woman's support system is encouraged to stay until discharge.

ANS: D Sleeping equipment is provided in a private room. A hospital setting is never a familiar environment to new parents. An LDRP room is not less expensive than a traditional hospital room. The baby remains with the mother at all times and is not removed to the nursery for routine care or testing. The father or other designated members of the mother's support system are encouraged to stay at all times.

10. A patient has delivered twins. The first twin was stillborn, and the second is in the intensive care nursery, recovering quickly from respiratory distress. The patient is crying softly and says, "I wish my baby could have lived." What is the most therapeutic response? a. "How soon do you plan to have another baby?" b. "Don't be sad. At least you have one healthy baby." c. "I have a friend who lost a twin and she's doing just fine now." d. "I am so sorry about your loss. Would you like to talk about it?"

ANS: D The nurse should recognize the woman's grief and its significance. Asking her about plans for another baby is denying the loss of the other infant. Pointing out the health of another baby is belittling her feelings. Stating that the nurse has a friend who lost a twin is denying the loss of the infant and her grief and belittling her feelings.

5. The upper uterus is the best place for the fertilized ovum to implant due to which anatomical adaptation? a. Maternal blood flow is lower. b. Placenta attaches most firmly. c. Uterine endometrium is softer. d. Developing baby is best nourished.

ANS: D The uterine fundus is richly supplied with blood and has the thickest endometrium, both of which promote optimum nourishment of the fetus. The blood supply is rich in the fundus, which allows for optimal nourishment of the fetus. If the placenta attaches too deeply, it does not easily detach. Softness is not a concern with implantation; attachment and nourishment are the major concerns.

2. What is the rationale for a woman in her first trimester of pregnancy to expect to visit her health care provider every 4 weeks? a. Problems can be eliminated. b. She develops trust in the health care team. c. Her questions about labor can be answered. d. The conditions of the expectant mother and fetus can be monitored

ANS: D This routine allows for monitoring maternal health and fetal growth and ensures that problems will be identified early. All problems cannot be eliminated because of prenatal visits; however, they can be identified early. Developing a trusting relationship should be established during these visits, but that is not the primary reason. Most women do not have questions concerning labor until the last trimester of the pregnancy

3. In order to increase the absorption of iron by a pregnant patient, which beverage should an iron preparation be given with? a. Tea b. Milk c. Coffee d. Orange juice

ANS: D Vitamin C source may increase the absorption of iron and would be the optimal choice. Tannin in the tea reduces the absorption of iron. The calcium and phosphorus in milk decrease iron absorption. Decreased intake of caffeine is recommended during pregnancy.

15. Which patient would require additional calories and nutrients? a. A 36-year-old female gravida 2, para 1, in her first trimester of pregnancy b. An 18-year-old female who delivered a 7-lb baby and is bottle feeding c. A 23-year-old female who had a cesarean birth and is bottle feeding d. A 20-year-old female who had a vaginal birth 5 months ago and is breastfeeding

ANS: D A patient who is breastfeeding will require more calories and nutrients than women who are pregnant. The type of birth has no impact on nutrient intake. A patient who is bottle-feeding does not require additional calories.

9. A placenta previa when the placental edge just reaches the internal os is called a. total. b. partial. c. low-lying. d. marginal.

ANS: D A placenta previa that does not cover any part of the cervix is termed marginal. With a total placenta previa, the placenta completely covers the os. With a partial previa, the lower border of the placenta is within 3 cm of the internal cervical os but does not completely cover the os. A complete previa is termed total. The placenta completely covers the internal cervical os.

11. When documenting a patient encounter, which term will the nurse use to describe the woman who is in the 28th week of her first pregnancy? a. Multigravida b. Multipara c. Nullipara d. Primigravida

ANS: D A primigravida is a woman pregnant for the first time. A multigravida has been pregnant more than once. A nullipara is a woman who has never been pregnant or has not completed a pregnancy of 20 weeks or more. A primipara has delivered one pregnancy of at least 20 weeks. A multipara has delivered two or more pregnancies of at least 20 weeks.

18. Which comment made by a patient in her first trimester indicates ambivalent feelings? a. "My body is changing so quickly." b. "I haven't felt well since this pregnancy began." c. "I'm concerned about the amount of weight I've gained." d. "I wanted to become pregnant, but I'm scared about being a mother."

ANS: D Ambivalence refers to conflicting feelings. Expressing a concern about being a mother indicates possible ambivalent feelings. Not feeling well since the pregnancy began does not reflect conflicting feelings. The woman is trying to confirm the pregnancy when she is stating the rapid changes to her body. She is not expressing conflicting feelings. By expressing concerns over gaining weight, which is normal, the woman is trying to confirm the pregnancy.

13. The nurse is explaining the function of the placenta to a pregnant patient. Which statement indicates to the nurse that further clarification is necessary? a. "My baby gets oxygen from the placenta." b. "The placenta functions to help excrete waste products." c. "The nourishment that I take in passes through the placenta." d. "The placenta helps maintain a stable temperature for my baby."

ANS: D Amniotic fluid and not the placenta helps with thermoregulation. The remaining statements are correct regarding placental function.

1. Which suggestion is most helpful for the pregnant patient who is experiencing heartburn? a. Drink plenty of fluids at bedtime. b. Eat only three meals a day so the stomach is empty between meals. c. Drink coffee or orange juice immediately on arising in the morning. d. Use Tums or Rolaids to obtain relief, as directed by the health care provider

ANS: D Antacids high in calcium (e.g., Tums, Rolaids) can provide temporary relief. Fluids overstretch the stomach and may precipitate reflux when lying down. Instruct the patient to eat five or six small meals per day rather than three full meals. Coffee and orange juice stimulate acid formation in the stomach and may need to be eliminated from the diet.

2. A pregnant woman has come to the emergency department with complaints of nasal congestion and epistaxis. Which is the correct interpertation of these symptoms by the health care provider? a. Nasal stuffiness and nosebleeds are caused by a decrease in progesterone. b. These conditions are abnormal. Refer the patient to an ear, nose, and throat specialist. c. Estrogen relaxes the smooth muscles in the respiratory tract, so congestion and epistaxis are within normal limits. d. Estrogen causes increased blood supply to the mucous membranes and can result in congestion and nosebleeds.

ANS: D As capillaries become engorged, the upper respiratory tract is affected by the subsequent edema and hyperemia, which causes these conditions, seen commonly during pregnancy. Progesterone is responsible for the heightened awareness of the need to breathe in pregnancy. Progesterone levels increase during pregnancy. The patient should be reassured that these symptoms are within normal limits. No referral is needed at this time. Relaxation of the smooth muscles in the respiratory tract is affected by progesterone.

1. A nurse educator is teaching a group of nursing students about the history of family-centered maternity care. Which statement should the nurse include in the teaching session? a. The Sheppard-Towner Act of 1921 promoted family-centered care. b. Changes in pharmacologic management of labor prompted family-centered care. c. Demands by physicians for family involvement in childbirth increased the practice of family-centered care. d. Parental requests that infants be allowed to remain with them rather than in a nursery initiated the practice of family-centered care.

ANS: D As research began to identify the benefits of early, extended parent-infant contact, parents began to insist that the infant remain with them. This gradually developed into the practice of rooming-in and finally to family-centered maternity care. The Sheppard-Towner Act provided funds for state-managed programs for mothers and children but did not promote family-centered care. The changes in pharmacologic management of labor were not a factor in family-centered maternity care. Family-centered care was a request by parents, not physicians.

9. A patient in her third trimester of pregnancy is asking about safe travel. Which statement should the nurse provide regarding safe travel during pregnancy? a. "Only travel by car during pregnancy." b. "Avoid use of the seat belt during the third trimester." c. "You can travel by plane until your 38th week of gestation." d. "If you are traveling by car stop to walk every 1 to 2 hours."

ANS: D Car travel is safe during normal pregnancies. Suggest that the woman stop to walk every 1 to 2 hours so she can empty her bladder. Walking also helps decrease the risk of thrombosis that is elevated during pregnancy. Seat belts should be worn throughout the pregnancy. Instruct the woman to fasten the seat belt snugly, with the lap belt under her abdomen and across her thighs and the shoulder belt in a diagonal position across her chest and above the bulge of her uterus. Travel by plane is generally safe up to 36 weeks if there are no complications of the pregnancy, so only traveling by car is an inaccurate statement.

14. In which situation would a dilation and curettage (D&C) be indicated? a. Complete abortion at 8 weeks b. Incomplete abortion at 16 weeks c. Threatened abortion at 6 weeks d. Incomplete abortion at 10 weeks

ANS: D D&C is carried out to remove the products of conception from the uterus and can be performed safely until week 14 of gestation. If all the products of conception have been passed (complete abortion), a D&C is not necessary. If the pregnancy is still viable (threatened abortion), a D&C is not indicated.

13. Which physiologic adaptation of pregnancy may lead to increased constipation during the pregnancy? a. Increased emptying time in the intestines b. Abdominal distention and bloating c. Decreased absorption of water d. Decreased motility in the intestines

ANS: D Decreased motility in the intestines leading to increased water absorption would cause constipation. Increased emptying time in the intestines leads to increased nutrient absorption. Abdominal distention and bloating are a result of increased emptying time in the intestines. Decreased absorption of water would not cause constipation.

10. Which is the most important reason for evaluating the pattern of weight gain in pregnancy? a. Prevents excessive adipose tissue deposits b. Determines cultural influences on the woman's diet c. Assesses the need to limit caloric intake in obese women d. Identifies potential nutritional problems or complications of pregnancy

ANS: D Deviations from the recommended pattern of weight gain may indicate nutritional problems or developing complications. Excessive adipose tissue may occur with excess weight gain but is not the reason for monitoring the weight gain pattern. The pattern of weight gain is not affected by cultural influences. It is important to monitor the pattern of weight gain for the developing complications.

10. What is the purpose of amniocentesis for a patient hospitalized at 34 weeks of gestation with pregnancy-induced hypertension? a. Determine if a metabolic disorder exists. b. Identify the sex of the fetus. c. Identify abnormal fetal cells. d. Determine fetal lung maturity

ANS: D During the third trimester, amniocentesis is most often performed to determine fetal lung maturity. In cases of pregnancy-induced hypertension, preterm birth may be necessary because of changes in placental perfusion. The test is done in the early portion of the pregnancy if a metabolic disorder is genetic. Amniocentesis is done early in the pregnancy to do genetic studies and determine the sex. Identification of abnormal cells is done during the early portion of the pregnancy.

23. A patient with no prenatal care delivers a healthy male infant via the vaginal route, with minimal blood loss. During the labor period, vital signs were normal. At birth, significant maternal hypertension is noted. When the patient is questioned, she relates that there is history of heart disease in her family; but, that she has never been treated for hypertension. Blood pressure is treated in the hospital setting and the patient is discharged. The patient returns at her scheduled 6-week checkup and is found to be hypertensive. Which type of hypertension is the patient is exhibiting? a. Pregnancy-induced hypertension (PIH) b. Gestational hypertension c. Preeclampsia superimposed on chronic hypertension d. Undiagnosed chronic hypertension

ANS: D Even though the patient has no documented prenatal care or medical history, she does relate a family history that is positive for heart disease. Additionally, the patient's blood pressure increased following birth and was treated in the hospital and resolved. Now the patient appears at the 6-week checkup with hypertension. Typically, gestational hypertension resolves by the end of the 6-week postpartum period. The fact that this has not resolved is suspicious for undiagnosed chronic hypertension. There is no evidence to suggest that the patient was preeclamptic prior to the birth.

12. A patient with preeclampsia is admitted complaining of pounding headache, visual changes, and epigastric pain. Nursing care is based on the knowledge that these signs indicate a. gastrointestinal upset. b. effects of magnesium sulfate. c. anxiety caused by hospitalization. d. worsening disease and impending convulsion.

ANS: D Headache and visual disturbances are caused by increased cerebral edema. Epigastric pain indicates distention of the hepatic capsules and often warns that a convulsion is imminent. Gastrointestinal upset is not an indication as severe as the headache and visual disturbance. She has not yet been started on magnesium sulfate as a treatment. The signs and symptoms do not describe anxiety.

7. Which goal is most appropriate for the collaborative problem of wound infection? a. The patient will not exhibit further signs of infection. b. Maintain the patient's fluid intake at 1000 mL/8 hour. c. The patient will have a temperature of 98.6F within 2 days. d. Monitor the patient to detect therapeutic response to antibiotic therapy.

ANS: D In a collaborative problem, the goal should be nurse-oriented and reflect the nursing interventions of monitoring or observing. Monitoring for complications such as further signs of infection is an independent nursing role. Intake and output is an independent nursing role. Monitoring a patient's temperature is an independent nursing role

17. Which clinical finding is associated with inadequate maternal weight gain during pregnancy? a. Prolonged labor b. Preeclampsia c. Gestational diabetes d. Low-birth-weight infant

ANS: D Inadequate maternal weight gain during pregnancy can manifest in the birth of a low-birth-weight infant. Prolonged labor and gestational diabetes are associated with excess weight gain during pregnancy. Preeclampsia is based on maternal hypertension, proteinuria, and edema states.

8. The patient has just learned that she is pregnant and overhears the gynecologist saying that she has a positive Chadwick's sign. When the patient asks the nurse what this means, how would the nurse respond? a. "Chadwick's sign signifies an increased risk of blood clots in pregnant women because of a congestion of blood." b. "That sign means the cervix has softened as the result of tissue changes that naturally occur with pregnancy." c. "This means that a mucus plug has formed in the cervical canal to help protect you from uterine infection." d. "This sign occurs normally in pregnancy, when estrogen causes increased blood flow in the area of the cervix."

ANS: D Increasing levels of estrogen cause hyperemia (congestion with blood) of the cervix, resulting in the characteristic bluish purple color that extends to include the vagina and labia. This discoloration, referred to as Chadwick's sign, is one of the earliest signs of pregnancy. Although Chadwick's sign occurs with hyperemia (congestion with blood), the sign does not signify an increased risk of blood clots. The softening of the cervix is called Goodell's sign, not Chadwick's sign. Although the formation of a mucus plug protects from infection, it is not called Chadwick's sign.

8. Which nursing intervention is written correctly? a. Force fluids as necessary. b. Observe interaction with the infant. c. Encourage turning, coughing, and deep breathing. d. Assist to ambulate for 10 minutes at 8 am, 2 pm and 6 PM

ANS: D Interventions might not be carried out if they are not detailed and specific. "Force fluids" is not specific; it does not state how much or how often. Encouraging the patient to turn, cough, and breathe deeply is not detailed or specific. Observing interaction with the infant does not state how often this procedure should be done. Assisting the patient to ambulate for 10 minutes within a certain timeframe is specific.

25. A pregnant patient with significant iron-deficiency anemia is prescribed iron supplements. The patient explains to the nurse that she cannot take iron because it makes her nauseous. What is the best response by the nurse? a. "Iron will be absorbed more readily if taken with orange juice." b. "It is important to take this drug regardless of this side effect." c. "Taking the drug with milk may decrease your symptoms." d. "Try taking the iron at bedtime on an empty stomach."

ANS: D Iron taken at bedtime may be easier to tolerate. All the answers are true statements; however, only the option that states that iron taken at bedtime may be easier to tolerate addresses both optimal absorption of iron and alleviation of nausea, which will not be noticeable during sleep. It is true that taking iron with milk will decrease the symptoms; however, it will also decrease absorption.

25. Which comment made by a new mother exhibits understanding of her toddler's response to a new sibling? a. "I can't believe he is sucking his thumb again." b. "He is being difficult and I don't have time to deal with him." c. "When we brought the baby home, we made Michael stop sleeping in the crib." d. "My husband is going to stay with the baby so I can take Michael to the park tomorrow."

ANS: D It is important for a mother to seek time alone with her toddler to reassure him that he is loved. It is normal for a child to regress when a new sibling is introduced into the home. The toddler may have feelings of jealousy and resentment toward the new baby taking attention away from him. Frequent reassurance of parental love and affection is important. Changes in sleeping arrangements should be made several weeks before the birth so the child does not feel displaced by the new baby.

7. A patient who is 16 weeks pregnant with her first baby asks how long it will be before she feels the baby move. Which is the nurse's best answer? a. "You should have felt the baby move by now." b. "The baby is moving, but you can't feel it yet." c. "Some babies are quiet and you don't feel them move." d. "Within the next month you should start to feel fluttering sensations."

ANS: D Maternal perception of fetal movement (quickening) usually begins between 17 and 20 weeks after conception. Because this is her first pregnancy, movement is felt toward the later part of the 17 to 20 weeks. "The baby is moving, but you can't feel it yet" may be alarming to the woman. "Some babies are quiet and you don't feel them move" is a true statement; the fetus' movements are not strong enough to be felt until 17 to 20 weeks; however, this statement does not answer the woman's concern. Fetal movement should be felt between 17 and 20 weeks; if movement is not perceptible by the end of that time, further assessment will be necessary.

5. A patient who is older than 35 years may have difficulty achieving pregnancy because a. prepregnancy medical attention is lacking. b. personal risk behaviors influence fertility. c. contraceptives have been used for an extended period of time. d. the ovaries may be affected by the normal aging process.

ANS: D Once the mature woman decides to conceive, a delay in becoming pregnant may occur because of the normal aging of the ovaries. Prepregnancy medical care is available and encouraged. The older adult participates in fewer risk behaviors than the younger adult. The problem is the age of the ovaries, not the past use of contraceptives. Fertility begins to decline at age 32 and decreases more rapidly by age 37.

11. Which action is the most appropriate nursing measure when a baby has an unexpected defect at birth? a. Remove the baby from the delivery area immediately. b. Inform the parents immediately that something is wrong. c. Tell the parents that the baby has to go to the nursery immediately. d. Explain the defect and show the baby to the parents as soon as possible.

ANS: D Parents experience less anxiety when they are told about the defect as early as possible and are allowed to touch and hold the baby. The parents should be able to touch and hold the baby as soon as possible. The nurse should not take the baby away; this would raise anxiety levels of the parents. They should be told about the defect and allowed to see the baby

15. Which of the patient health behaviors in the first trimester would the nurse identify as a risk factor in pregnancy? a. Sexual intercourse two or three times weekly b. Moderate exercise for 30 minutes daily c. Working 40 hours a week as a secretary in a travel agency d. Relaxing in a hot tub for 30 minutes a day, several days a week

ANS: D Pregnant women should avoid activities that might cause hyperthermia. Maternal hyperthermia, particularly during the first trimester, may be associated with fetal anomalies. She should not be in a hot tub for more than 10 minutes at less than 100F. Sexual intercourse is generally safe for the healthy pregnant woman; moderate exercise during pregnancy can strengthen muscles, reduce backache and stress, and provide a feeling of well-being; working during pregnancy is acceptable as long as the woman is not continually on her feet or exposed to environmental toxins and industrial hazards.

29. The nurse is reviewing a list of foods high in folic acid with a patient who is considering becoming pregnant. The nurse determines that the patient understands the teaching when the patient states she will include which list of foods in her diet? a. Peaches, yogurt, and tofu b. Strawberries, milk, and tuna c. Asparagus, lemonade, and chicken breast d. Spinach, orange juice, and fortified bran flakes

ANS: D Prepregnant, the recommendation for folic acid is 800 mcg. Foods high in folic acid are dark green leafy vegetables, legumes (beans, peanuts), orange juice, asparagus, spinach, and fortified cereal and pasta. In the United States, folic acid is added to orange juice and wheat-based products.

14. Which physiologic findings related to gallbladder function may lead to the development of gallstones during pregnancy? a. Decrease in alkaline phosphatase levels compared with nonpregnant women b. Increase in albumin and total protein as a result of hemodilution c. Hypertonicity of gallbladder tissue d. Prolonged emptying time

ANS: D Prolonged emptying time is seen during pregnancy and may lead to the development of gallstones. In pregnancy, there is a twofold to fourfold time increase in alkaline phosphatase levels as compared with those in nonpregnant woman. During pregnancy, a decrease in albumin level and total protein is seen as a result of hemodilution. Gallbladder tissue becomes hypotonic during pregnancy.

24. A relaxation technique that can be used during the childbirth experience to decrease maternal pain perception is a. using increased environmental stimulation as a method of distraction. b. restricting family and friends from visiting during the labor period to keep the patient focused on breathing techniques. c. medicating the patient frequently to reduce pain perception. d. assisting the patient in breathing methods aimed at taking control of pain perception based on the contraction pattern.

ANS: D Relaxation techniques are aimed at incorporating mind and body activities to maintain control over pain. Additional environmental stimuli may have the opposite effect and increase patient anxiety, which will affect pain perception. Restricting visitors may have the opposite effect, leading to increased anxiety because of isolation. Medicating a patient may not decrease pain perception but may place the patient at risk for adverse reactions and/or complications of pregnancy related to medications.

6. A pregnant patient's diet may not meet her need for folate. Which food choice is an excellent source of this nutrient? a. Chicken b. Cheese c. Potatoes d. Green leafy vegetables

ANS: D Sources of folate include green leafy vegetables, whole grains, fruits, liver, dried peas, and beans. Chicken is a good source of protein, but poor in folate. Cheese is an excellent source of calcium, but poor in folate. Potatoes contain carbohydrates and vitamins but are poor in folate.

13. What is the purpose of initiating contractions in a contraction stress test (CST)? a. Increase placental blood flow. b. Identify fetal acceleration patterns. c. Determine the degree of fetal activity. d. Apply a stressful stimulus to the fetus.

ANS: D The CST involves recording the response of the fetal heart rate to stress induced by uterine contractions. The CST records the fetal response to stress. It does not increase placental blood flow. The NST looks at fetal heart accelerations with fetal movements. The NST and biophysical profiles look at fetal movements.

21. Which is the method of childbirth that helps prevent the fear-tension-pain cycle by using slow abdominal breathing in early labor and rapid chest breathing in advanced labor? a. Bradley b. Lamaze c. Leboyer d. Dick-Read

ANS: D The Dick-Read method helps prevent the fear-tension-pain cycle by using slow abdominal breathing in early labor and rapid chest breathing in advanced labor. The Lamaze method involves concentration and conditioning to help the woman respond to contractions with relaxation to decrease pain. Viewing childbirth as a traumatic experience, the Leboyer method uses decreased light and noise to help the newborn adapt to extrauterine life more easily. The Bradley method teaches women to use abdominal muscles to increase relaxation and breath control; it emphasizes avoidance of all medications and interventions.

23. What does a birth plan help the parents accomplish? a. Avoidance of an episiotomy b. Determining the outcome of the birth c. Assuming complete control of the situation d. Taking an active part in planning the birth experience

ANS: D The birth plan helps the woman and her partner look at the available options and plan the birth experience to meet their personal needs. A birth plan cannot dictate the need for or avoidance of an episiotomy. The outcome of the birth is not an absolute determinant. A birth plan does not assume complete control of the situation; it allows for expanding communication. Parents who prepare a birth plan should be educated that flexibility is essential as each labor and delivery is unique and may present unexpected complications.

4. The nurse is explaining the process of cell division during the preembryonic period to a group of nursing students. Which statement best describes the characteristics of the morula? a. Fertilized ovum before mitosis begins b. Double layer of cells that becomes the placenta c. Flattened, disk-shaped layer of cells within a fluid-filled sphere d. Solid ball composed of the first cells formed after fertilization

ANS: D The morula is so named because it resembles a mulberry. It is a solid ball of 12 to 16 cells that develops after fertilization. The fertilized ovum is called the zygote. The placenta is formed from two layers of cells—the trophoblast, which is the other portion of the fertilized ovum, and the decidua, which is the portion of the uterus where implantation occurs. The flattened, disk-shaped layer of cells is the embryonic disk; it will develop into the body

11. Which nursing diagnosis should the nurse identify as a priority for a patient in active labor? a. Risk for anxiety related to upcoming birth b. Risk for imbalanced nutrition related to NPO status c. Risk for altered family processes related to new addition to the family d. Risk for injury (maternal) related to altered sensations and positional or physical changes

ANS: D The nurse should determine which problem needs immediate attention. Risk for injury is the problem that has the priority at this time because it is a safety problem. Risk for anxiety, imbalanced nutrition, and altered family processes are not the priorities at this time.

6. Which finding is a positive sign of pregnancy? a. Amenorrhea b. Breast changes c. Fetal movement felt by the woman d. Visualization of fetus by ultrasound

ANS: D The only positive signs of pregnancy are auscultation of fetal heart tones, visualization of the fetus by ultrasound, and fetal movement felt by the examiner. Amenorrhea is a presumptive sign of pregnancy. Breast changes are a presumptive sign of pregnancy. Fetal movement is a presumptive sign of pregnancy.

10. The nurse is writing an expected outcome for the nursing diagnosis—acute pain related to tissue trauma, secondary to vaginal birth, as evidenced by patient stating pain of 8 on a scale of 10. Which expected outcome is correctly stated for this problem? a. Patient will state that pain is a 2 on a scale of 10. b. Patient will have a reduction in pain after administration of the prescribed analgesic. c. Patient will state an absence of pain 1 hour after administration of the prescribed analgesic. d. Patient will state that pain is a 2 on a scale of 10, 1 hour after the administration of the prescribed analgesic.

ANS: D The outcome should be patient-centered, measurable, realistic, and attainable and within a specified timeframe. Patient stating that her pain is now 2 on a scale of 10 lacks a timeframe. Patient having a reduction in pain after administration of the prescribed analgesic lacks a measurement. Patient stating an absence of pain 1 hour after the administration of prescribed analgesic is unrealistic.

17. A patient reports to the clinic nurse that she has not had a period in over 12 weeks, she is tired, and her breasts are sore all of the time. The patient's urine test is positive for hCG. What is the correct nursing action related to this information? a. Ask the patient if she has had any nausea or vomiting in the morning. b. Schedule the patient to be seen by a health care provider within the next 4 weeks. c. Send the patient to the maternity screening area of the clinic for a routine ultrasound. d. Determine if there are any factors that might prohibit her from seeking medical care.

ANS: D The patient has presumptive and probable indications of pregnancy. However, she has not sought out health care until late in the first or early in the second trimester. The nurse must assess for barriers to seeking health care, physical or emotional, because regular prenatal care is key to a positive pregnancy outcome. Asking if the patient has nausea or vomiting will only add to the list of presumptive signs of pregnancy, and this information will not add to the assessment data to determine whether the patient is pregnant. The patient needs to see a health care provider before the next 4 weeks because she is late in seeking early prenatal care. Ultrasound testing must be prescribed by a health care provider.

27. A pregnant patient comes into the medical clinic stating that her family and friends are telling her that she is always talking about the pregnancy and nothing else. She is concerned that something is wrong with her. What psychological behavior is she exhibiting? a. Antepartum obsession b. Ambivalence c. Uncertainty d. Introversion

ANS: D The patient is exhibiting behaviors associated with introversion and/or narcissism. These are normal findings during pregnancy as long as they do not become obsessive to the exclusion of everything else. The patient is talking about the pregnancy but there is no evidence that it is affecting her perception of reality and/or ability to perform ADLs. It is normal for pregnant women to focus on the self as being of prime importance in their life initially during the pregnancy. Some women may feel ambivalent about their pregnancy, which is a normal reaction. However, this patient's behavior does not support this finding. Some women react with uncertainty at the news of being pregnant, which is a normal reaction. However, this patient's behavior does not support this finding.

26. A patient who had premature rupture of the membranes (PROM) earlier in the pregnancy at 28 weeks returns to the labor unit 1 week later complaining that she is now in labor. The labor and birth nurse performs the following assessments. The vaginal exam is deferred until the physician is in attendance. The patient is placed on electronic fetal monitoring (EFM) and a baseline FHR of 130 bpm is noted. No contraction pattern is observed. The patient is then transferred to the antepartum unit for continued observation. Several hours later, the patient complains that she does not feel the baby move. Examination of the abdomen reveals a fundal height of 34 cm. Muscle tone is no different from earlier in the hospital admission. The patient is placed on the EFM and no fetal heart tones are observed. What does the nurse suspect is occurring? a. Placental previa b. Active labor has started c. Placental abruption d. Hidden placental abruption

ANS: D The patient's signs and symptoms indicate that a hidden abruption is occurring. Fundal height has increased and there is an absence of fetal heart tones. This is a medical emergency and the physician should be contacted to come directly to the unit for intervention and imminent birth

25. Which technique would provide the best pain relief for a pregnant woman with an occiput posterior position? a. Neuromuscular disassociation b. Effleurage c. Psychoprophylaxis d. Sacral pressure

ANS: D The use of sacral pressure may provide relief for patients who are experiencing back labor. The presentation of the fetus in a posterior position indicates this. Neuromuscular dissociation is used as a conditioned response to affect pain relief based on the mother tensing one group of muscles and focusing on releasing tension in the rest of her body. Effleurage is the process of using circular massage to effect pain relief. Psychoprophylaxis is another name for the Lamaze method of prepared childbirth

16. A pregnant woman notices that she is beginning to develop dark skin patches on her face. She denies using any different type of facial products as a cleansing solution or makeup. What would the priority nursing intervention be in response to this situation? a. Refer the patient to a dermatologist for further examination. b. Ask the patient if she has been eating different types of foods. c. Take a culture swab and send to the lab for culture and sensitivity (C&S). d. Let the patient know that this is a common finding that occurs during pregnancy.

ANS: D This condition is known as chloasma or melasma (mask of pregnancy) and is a result of pigmentation changes relative to hormones. It can be exacerbated by exposure to the sun. There is no need to refer to a dermatologist. Intake of foods is not associated with exacerbation of this process. There is no need for a C&S to be taken. The patient should be assured that this is a normal finding of pregnancy.

13. Determine the obstetric history of a patient in her fifth pregnancy who has had two spontaneous abortions in the first trimester, one infant at 32 weeks' gestation, and one infant at 38 weeks' gestation. a. G5 T1 P2 A2 L 2 b. G5 T1 P1 A1 L2 c. G5 T0 P2 A2 L2 d. G5 T1 P1 A2 L2

ANS: D This patient is in her fifth pregnancy, which is G5, she had one viable term infant (between 38 and 42 weeks' gestation), which is T1, she had one viable preterm infant (between 20 and 37 weeks' gestation), which is P1, two spontaneous abortions (before 20 weeks' gestation), which is A2, and she has two living children, which is L2.

3. While providing education to a primiparous patient regarding the normal changes of pregnancy, what is an important information for the nurse to share regarding Braxton Hicks contractions? a. These contractions may indicate preterm labor. b. These are contractions that never cause any discomfort. c. Braxton Hicks contractions only start during the third trimester. d. These occur throughout pregnancy, but you may not feel them until the third trimester.

ANS: D Throughout pregnancy, the uterus undergoes irregular contractions called Braxton Hicks contractions. During the first two trimesters, the contractions are infrequent and usually not felt by the woman until the third trimester. Braxton Hicks contractions do not indicate preterm labor. Braxton Hicks contractions can cause some discomfort, especially in the third trimester. Braxton Hicks contractions occur throughout the whole pregnancy.

12. To determine cultural influences on a patient's diet, what is the nurse's primary action? a. Evaluate the patient's weight gain during pregnancy. b. Assess the socioeconomic status of the patient. c. Discuss the four food groups with the patient. d. Identify the food preferences and methods of food preparation common to the patient's culture.

ANS: D Understanding the patient's food preferences and how she prepares food will assist the nurse in determining whether the patient's culture is adversely affecting her nutritional intake. Evaluating a patient's weight gain during pregnancy should be included for all patients, not just for those who are culturally different. The socioeconomic status of the patients may alter the nutritional intake, but not the cultural influence. Teaching the food groups to the patient should come after assessing food preferences.

26. An expectant couple asks the nurse about intercourse during pregnancy and whether it is safe for the baby. What information should the nurse provide? a. Intercourse is safe until the third trimester. b. Safer sex practices should be used once the membranes rupture. c. Intercourse should be avoided if any spotting from the vagina occurs afterward. d. Intercourse and orgasm are often contraindicated if a history of or signs of preterm labor are present.

ANS: D Uterine contractions that accompany orgasm can stimulate labor and would be problematic if the woman is at risk for or has a history of preterm labor. Intercourse can continue as long as the pregnancy is progressing normally. Rupture of the membranes may require abstaining from intercourse. Safer sex practices are always recommended. Some spotting can normally occur as a result of the increased fragility and vascularity of the cervix and vagina during pregnancy

46. A patient, who delivered her third child yesterday, has just learned that her two school-age children have contracted chickenpox. What should the nurse tell her? a. Her two children should be treated with acyclovir before she goes home from the hospital. b. The baby will acquire immunity from her and will not be susceptible to chickenpox. c. The children can visit their mother and baby in the hospital as planned but must wear gowns and masks. d. She must make arrangements to stay somewhere other than her home until the children are no longer contagious.

ANS: D Varicella (chickenpox) is highly contagious. Although the baby inherits immunity from the mother, it would not be safe to expose either the mother or the baby. Acyclovir is used to treat varicella pneumonia. The baby is already born and has received the immunity. If the mother never had chickenpox, she cannot transmit the immunity to the baby. Varicella infection occurring in a newborn may be life-threatening

4. When should iron supplementation during a normal pregnancy begin? a. Before pregnancy b. In the first trimester c. In the third trimester d. In the second trimester

ANS: D Vitamin supplements should be prescribed in the second trimester, when the need for iron is increased. Healthy young women do not usually need iron supplementation for their diets. Morning sickness in the first trimester increases the routine side effects of iron supplements. The iron supplements may continue to be prescribed in the third trimester and during the postpartum period.

5. The RN is delegating tasks to the unlicensed assistive personnel (UAP). Which tasks can the nurse delegate? (Select all that apply.) a. Teaching the patient about breast care b. Assessment of a patient's lochia and perineal area c. Assisting a patient to the bathroom for the first time after birth d. Vital signs on a postpartum patient who delivered the night before e. Assisting a postpartum patient to take a shower on the second postpartum day

ANS: D, E Nurses must be aware that they remain legally responsible for patient assessments and must make the critical judgments necessary to ensure patient safety when delegating tasks to unlicensed personnel. The nurse cannot delegate assessment, teaching, or evaluation. The two tasks that the nurse can delegate are vital signs on a stable postpartum patient and assisting a stable postpartum patient on the second postpartum day to take a shower.


Related study sets

POLS 2401: Key Actors & Ideas on the World Stage

View Set

Behavioral - Block 2 Practice Questions

View Set

Endangered Species: The African elephant

View Set

2.3 Summarize secure application development, deployment, and automation concepts

View Set

Quiz 1 Male/Female Reproductive System/Disorder

View Set